Download as pdf or txt
Download as pdf or txt
You are on page 1of 60

Genodermatoses

1. Which of the following first-line therapeutic modalities is INCORRECTLY matched with the vascular
malformation?
A. Arteriovenous Malformation and Excision
B. Venous Malformation and Sclerotherapy
C. Telangiectasia and Pulsed Dye Laser
D. Lymphatic Malformation and Sclerotherapy
E. Capillary Malformation and Pulsed Dye Laser

Correct answer: (A) Arteriovenous Malformation and Excision.


Explanation: All of the choices, (B) (C) (D) (E), are correctly matched with their first-line therapy except for Choice
(A). Embolization, which is the delivery of an embolic agent into the AVM in order to block blood flow and decrease
shunting, is usually the first-line therapy for Arteriovenous Malformations. More invasive interventions, such as
surgical excisions, can worsen lesions secondary to trauma. If necessary, surgical management of AVM’s requires
extreme caution and skilled surgeons in order to prevent exacerbation, recurrence, or compromise the patient.
While relatively novel, sclerotherapy has emerged as a first-line therapy for lymphatic malformations, both
microcystic and macrocystic. Similar to venous malformations, sclerotherapy can be performed relatively early in
patients, with neonates 6 months of age safely undergoing the procedure. Telangiectasias, a smaller version of
capillary malformations, are very responsive to Pulsed Dye Laser.

2. What is the gene mutation responsible for this skin disease?

A. Keratin 5
B. Keratin 14
C. Type VII Collagen
D. Laminin 5
E. Plectin

Correct answer: (C) Type VII Collagen.


Explanation: Type VII Collagen gene mutation is responsible for Dystrophic EpidermolysisBullosa. The main
features of the Hallopeau-Siemens (recessive) subtype seen in the above picture include “mitten deformity” of
hands/feet, severe widespread bullae at birth that heal with atrophic scarring, nail dystrophy, mucous membrane
involvement, dysplastic teeth, milia, and squamous cell carcinoma.
(A) Keratin 5 is mutated in EpidermolysisBullosa Simplex and Dowling-Degos disease.
(B) Keratin 14 is mutated in EpidermolysisBullosa Simplex.
(D) Laminin 5 mutation is seen in JunctionalEpidermolysisBullosa, and Herlitz Type.
(E) Plectin mutation is seen in EpidermolysisBullosa Simplex with Muscular Dystrophy.

1
3. Which of the following associations is the earliest manifestation of Neurofibromatosis Type 1?
A. Optic Glioma
B. Cutaneous Neurofibroma
C. Sphenoid Wing Dysplasia
D. Café Au Lait Macules
E. Axillary Freckling

Correct answer: (C) Sphenoid Wing Dysplasia.


Explanation: Both plexiformneurofibromas and skeletal defects, such as sphenoid wing dysplasia, have a greater
propensity for being congenital and present at birth than any of the other listed findings. While skeletal defects
may not present clinically, they are nonetheless more likely to be the earliest manifestation.
(A) (E) Ocular findings such as Lisch Nodules and Optic Gliomas do not typically appear until children are school
aged, and a similar onset in timing has been observed for intertriginous freckling.
(B) Cutaneous neurofibromas do not typically appear until children are near puberty.
(D) While CALMs may sometimes be congenital, a majority of café au lait macules do not present until the first few
years of life. If the question had asked, which of the following is the earliest cutaneous manifestation associated
with NF-1, CALMs would be the correct answer. By 8 years of age, 97% of affected individuals have the necessary
criteria for the diagnosis of NF-1, with 100% of patients meeting the criteria by age 20.

4. Which of the following statements regarding Neurofibromatosis Type 1 is true?


A. Diagnostic criteria for Café Au Lait Macules requires a measurement greater than 15mm in
children less then 10 years of age
B. Diagnostic criteria for Café Au Lait Macules requires a measurement of less then 0.2 cm in
children less then 10 years of age
C. Diagnostic criteria for Café Au Lait Macules requires a measurement greater then 0.5 mm in
post-pubertal children
D. Diagnostic criteria for Café Au Lait Macules requires a measurement of greater then or equal
to 5 mm in pre-pubertal children
E. Diagnostic criteria for Café Au Lait Macules requires a measurement of greater than or equal
to 2.0 cm in post-pubertal children

Correct answer: (D) Diagnostic criteria for Café Au Lait Macules requires a measurement of greater then or
equal to 5 mm in pre-pubertal children.
Explanation: The diagnostic criteria for Neurofibromatosis Type 1, requires 6 or more Café Au Lait Macules
(CALMs). In pre-pubertal children the size must be greater than 0.5 cm (5mm), while in post-pubertal adolescents
the size criteria for CALMs is greater than 1.5 cm (15mm). Puberty varies from child to child and therefore no
specific age is set to determine the size restrictions for inclusion as criteria.
(A) (B) (C) (E) These are incorrect.

5. What does the Dyskerin (DKC1) gene encode?


A. Protein tyrosine phosphatase
B. Tyrosine kinase receptor
C. Telomerase complex
D. DNA Helicase
E. Nuclear envelope protein

Correct answer: (C) Telomerase complex.


Explanation: Telomerase complex is encoded by the Dyskerin (DKC1) gene which is altered in X-Linked
DyskeratosisCongenita, also known as Zinsser-Engman-Cole Syndrome. It is characterized by reticulated gray

2
brown hyperpigmentation / telangiectasias / atrophy / hypopigmentation, alopecia, premalignant leukoplakia of
any mucosal surface, pancytopenia, mild intellectual disability and dystrophic nails. Autosomal dominant
DyskeratosisCongenitais caused by mutations in TERC, which is involved in a telomerase RNA component.
(A) Protein tyrosine phosphatase is encoded by PTPN11 and is altered in Noonan Syndrome. Noonan syndrome is
characterized by autosomal dominant inheritance, low-set ears, webbed neck, short stature, lymphedema,
pigmented nevi, cafe au lait macules, hypertelorism, low-set ears, high-arched palate, pectusexcavatum and
intellectual disability.
(B) Tyrosine kinase receptor is encoded by the RET proto-oncogene and is mutated in Multiple Endocrine Neoplasia
Type II.
(D) DNA Helicase is encoded by RecQL4 and mutations are seen in Rothmund-Thompson Syndrome. Rothmund-
Thompson syndrome, also known as poikilodermacongenitale, is characterized by autosomal recessive inheritance,
poikiloderma on face/buttocks/extremities, photosensitivity, alopecia, short stature, cataracts, hypoplastic
thumbs, hypogonadism, premalignant acral keratosis, nail dystrophy, increased risk of squamous cell carcinoma,
osteosarcoma and other skeletal abnormalities like absent radii.
(E) Nuclear envelope protein mutation in Lamin A has been linked to Progeria. Progeria is characterized by
autosomal dominant inheritance, premature aging, beaked nose, front bossing, scleroderma-like skin, vein
prominence of scalp, alopecia, short stature, failure to thrive, atherosclerosis, high-pitched voice and premature
death.

6. All of the following are diagnostic criteria for Neurofibromatosis Type 1 (NF-1) EXCEPT?
A. Thinning of long bone cortices
B. Inguinal freckling
C. A maternal Aunt with Neurofibromatosis Type 1
D. Iris hamartomas
E. Plexiformneurofibromas

Correct answer: (C) A maternal Aunt with Neurofibromatosis Type 1.


Explanation: To be considered a component of the diagnostic criteria for NF-1, the disease must be found in a first
degree relative (parent, sibling, child). Relatives outside of the immediate family, although positive for NF-1, do
not contribute to the criteria necessary for diagnosis.
(A) Thinning in the cortex of the long bones +/- pseudoarthroses, and sphenoid wing dysplasia are part of the
diagnostic criteria for NF-1.
(B) Intertriginous freckling, which can occur in the axillae or inguinal folds is also known as Crowe’s Sign. These
pigmented macules are similar histopathologically to lentigines and are another cutaneous diagnostic criteria.
(D) Iris Hamartomas, also known as Lisch Nodules, are benign growths visible on the iris and are also part of the
diagnostic criteria of NF-1. Two or more must be present. Optic gliomas, another ocular manifestation of NF-1, can
lead to blindness warranting ophthalmologic examination. This finding is a separate criteria for NF-1.
(E) The presence of a solitary plexiformneurofibroma, as well as 2 or more cutaneous neurofibromas are also
diagnostic criteria for NF-1. In Summary, two or more of the following seven criteria must be present for a
diagnosis of NF-1. 1. Six or more CALMs > 0.5 cm in a pre-pubertal patient, or greater than 1.5 cm in a post-
pubertal patient. 2. Two or more neurofibromas of any type or one plexiformneurofibroma. 3. Intertriginous
Freckling. 4. Optic Gliomas. 5. Two or more Lisch Nodules (Iris Hamartomas). 6. Bone lesions such as thinning of the
long bone cortex or sphenoid wing dysplasia. 7. Finally, a First-degree relative with NF-1 meeting the above
criteria.

3
7. A patient presents to your clinic with the findings illustrated below. She is concerned because her mother had
similar clinical findings and died prematurely due to a cancer that was never revealed to her family. The
patient has had numerous colonoscopies with removal of benign hamartomatous polyps. Which of the following
malignancies should the patient be made aware of a possible increased risk of acquiring?

A. Breast Carcinoma
B. Uterine Carcinoma
C. Pancreatic Carcinoma
D. Ovarian Carcinoma
E. All of the above

Correct answer: (E) All of the above.


Explanation: The vignette in question details several of the clinical findings seen in Peutz-Jeghers syndrome. In
addition to the oral melanotic macules, the patient’s history of repeated excisions of benign
hamartomatouspolyps, and the early death of her mother who also likely had the disease, this patient has an
increased risk for each of the malignancies in Choices (A), (B), (C) and (D). The inheritance of Peutz-Jeghers is
autosomal dominant and it is likely that the patient’s mother succumbed to one of the associated malignancies.
The risk of malignancy in patients with Peutz-Jeghers is 18 times greater than that of the general population. In
patients with a known diagnosis, confirmed by genetic testing, yearly examination for organs at risk of malignancy
should begin in early adulthood. This includes breast, ovarian, uterine, thyroid, pancreas, and in males, testes.

8. All of the following are examples of vascular malformations EXCEPT?


A. Pyogenic Granuloma
B. Angiokeratoma
C. Telangiectasia
D. Arteriovenous Fistula
E. Port-Wine Stain

Correct answer: (A) Pyogenic Granuloma.


Explanation: Pyogenic granulomas are a type of vascular tumor. The remaining choices, (B), (C), (D) and (E), are
examples of vascular malformations. It is important to be able to recognize the difference between vascular
tumors and vascular malformations as their long-term management as well as treatment vary greatly.
Differentiating vascular tumors from malformations is dependent on natural history, appearance on histology, and
cellular markers. Vascular malformations are composed of malformed and dysplastic vessels while a vascular
tumor is a collection of hyperplastic cells that are of vascular origin. An example of which would be a Pyogenic
Granuloma, a tumor composed of hyperplastic endothelial cells that on pathology demonstrate an extensive and
branching network of small capillaries.

4
9. A patient presents with malodorous palmoplantarkeratoderma, gingival involvement and a history of pyogenic
infections. What is the most likely condition?
A. Papillon-Lefevre Syndrome
B. Vohwinkel Syndrome
C. Howel-Evans Syndrome
D. Richner-Hanhart Syndrome
E. Darier Disease

Correct answer: (A) Papillon-Lefevre Syndrome.


Explanation: Papillon-Lefevre Syndrome is also known as palmoplantarkeratoderma with periodontosis and is
characterized by autosomal recessive inheritance of CTSC gene mutation encoding for cathepsin C. Key features
include malodorous plantar keratoderma with transgrediens, punctate keratoderma, periodontitis with severe
gingivitis, pyogenic infections and dural calcification.
(B) Vohwinkel Syndrome is characterized by autosomal dominant inheritance of GJB2 gene mutations encoding
connexin 26, a gap junction protein. Key features include pseudo-ainhum with starfish shaped keratotic plaques on
dorsal hand, alopecia and high frequency hearing loss. The variant form of Vohwinkel syndrome (mild
ichthyosis/erythroderma without hearing loss) sometimes called loricrinkeratoderma, is caused by mutations in
the LOR gene.
(C) Howel-Evans Syndrome is characterized by autosomal dominant inheritance of a mutation within the TOC gene
locus. Key features include non-transgredienspalmoplantarkeratoderma feet>hands, esophageal carcinoma, and
oral leukoplakia.
(D) Richner-Hanhart Syndrome is characterized by autosomal recessive inheritance, TAT (hepatic tyrosine
aminotransferase) gene mutation, painful focal hyperkeratosis on weight-bearing plantar surface,
hypothenar/thenar eminences, elbows/knees/fingertips, or diffuse palmoplantarkeratoderma and keratitis.
(E) Darier Disease is characterized by autosomal dominant inheritance of ATP2A2/SERCA gene mutations,
hyperkeratoticseborrheic distribution papules/plaques, acrokeratosisverruciformis, and red/white longitudinal
bands and V-shaped nicking of nails.

10. What condition is most likely seen in the following picture?

A. Monilethrix
B. ArginosuccinicAciduria
C. Menkes’ Syndrome
D. Uncombable Hair Syndrome
E. Bjornstad Syndrome

Correct answer: (A) Monilethrix.


Explanation: Monilethrix is is characterized by autosomal dominant inheritance, mutations in hair keratin genes,
elliptical nodes along the hair shaft giving a “beaded” appearance, keratosis pilaris, brittle nails, and teeth
abnormalities.
(B) ArginosuccinicAciduriais characterized by trichorrhexisnodosa.

5
(C) Menkes’ Syndrome is characterized usually by pilitorti (twisting of hair shaft). Trichorrhexisnodosa and
monilethrixare rarely seen.
(D) Uncombable Hair Syndrome is characterized by pilitriangulietcanaliculi.
(E) Bjornstad Syndrome is characterized by pilitorti (twisting of hair shaft) with/without alopecia of scalp.

11. In which genodermatoses are blue sclerae NOT a potential feature?


A. Marfan’s Syndrome
B. Waardenburg Syndrome
C. OsteogenesisImperfecta
D. All of above

Correct answer: (B) Waardenburg Syndrome.


Explanation: Blue scleraecan be seen in all of the above except in Waardenburg Syndrome. Waardenburg
syndrome is characterized by dystopia canthorum with heterochromiairidis.

12. When a genodermatosisis said to have an incomplete degree of penetrance, what does that mean?
A. That only females can be affected with few exceptions
B. That only males can be affected with few exceptions.
C. That not all individuals who carry the disease genotype will manifest the disease.
D. That the disease can show variation in the severity of the findings that develop.
E. That the inactivation of one of the X in women is responsible for dose compensation.

Correct answer: (C) That not all individuals who carry the disease genotype will manifest the disease.
Explanation: A genodermatosis is said to have incomplete (or reduced penetrance) when not all the individuals
who carry the disease genotype manifest the condition. The degree of penetrance indicates the probability that an
individual with the disease genotype develops the disease phenotype. Penetrance is an “all-or-nothing”
phenomenon, in which some individuals who have inherited the disease genotype do not express the disease
phenotype at all. The genodermatoses that can only affect women, with few exceptions are usually X-linked
dominant and fatal to male embryos. The genodermatoses that can only affect men with few exceptions are X-
linked recessive. Variability in expression is defined by the variation in the severity of the findings that develop
when the individual has the disease genotype. The process by which there is inactivation of one of the X in women
and that is responsible for dose compensation is calledlyonisation or X-inactivation. It occurs randomly in each cell
of the female with the inactivation of one of the two X chromosomes.

13. A missense mutation in the gene responsible for the pictured diagnosis results in what other inherited disease?

A. Ichthyosis Vulgaris
B. Lamellar ichthyosis
C. Refsum disease
D. X-linked ichthyosis

6
Correct answer: (B) Lamellar ichthyosis.
Explanation: The photograph shows a neonate with the characteristic features of harlequin ichthyosis– severe
ectropion and eclabium and thick yellow hyperkeratotic adherent plaques. Deletion mutations in ABCA12, an ATP-
binding cassette transporter involved in lipid transport is responsible for harlequin ichthyosis. Minor missense
mutations in ABCA12 have been reported to underlie a type of lamellar ichthyosis. Lamellar ichthyosis presents at
birth with a collodion membrane, which after the first weeks of life develops into large plate like scales. Ectropion
and eclabium can result. The most common mutation of lamellar ichthyosis is transglutaminase 1.
(A) Ichthyosis vulgaris is the most common ichthyosis and presents with scaling on the extensor surfaces. It results
from a mutation in filaggrin, a protein involved in the aggregation of intermediate filaments. Mutations in
filaggrin are also associated with atopic dermatitis.
(C) Refsum disease is a neurocutaneous disorder, presenting with ichthyosis but also weakness, retinitis
pigmentosa and a neuropathy in addition to other neurological symptoms which are progressive. It is caused by
mutations in phyatnoyl-CoA hydroxylase or peroxisome biogenesis factor 7 genes, which leads to the build up of
phytanic acid. Reduction of dietary phytanic acid (animal products) can improve neurological disease.
(D) X-linked ichthyosis results from mutations in steroid sulfatase, which results in impaired hydrolysis of
cholesterol. Absence of this enzyme in the fetal placenta causes inadequate estrogen synthesis which can result in
failure of the cervix to dilate and labor to progress. Mild diffuse scaling can eventually result in brownish-colored
plate-like adherent lesions that commonly involve the anterior aspect of the lower extremities. Males have a risk
of cryptorchidism and the development of testicular cancer.

14. A biopsy report sent back to you from your pathology lab indicates the tissue specimen sent, while conclusively
a vascular malformation, cannot be further characterized into a more specific type of vascular malformation
without further immunohistochemical stains. Which of the following stains is specific for lymphatic
malformations?
A. CD34
B. LYVE-1
C. VEGFR-1
D. PECAM-1
E. Collagen Type IV

Correct answer: (B) LYVE-1.


Explanation: In addition to LYVE-1 (Lymphatic vascular endothelial receptor – 1), Prox1, a protein necessary for
lymphatic growth and development, as well as podoplanin, a surface glycoprotein, are all specific markers for
lymphatic vessels and lymphatic malformations. LYVE-1, a CD44 homologue is specific to lymphatic endothelial
cells.
(A) (C) (E) CD34, Collagen Type IV and VEGFR-1 are immunohistochemical markers of the endothelial cells lining
capillaries, arteries and veins, but not lymphatic vessels.
(D) PECAM-1 (also known as CD31, platelet endothelial cell adhesion molecule 1), stains both lymphatics and blood
vessels and thusly would not be specific for lymphatic malformations.

15. Which of the following tooth finding is seen in Hyper IgE syndrome?
A. Pegged teeth
B. Retention of primary teeth
C. Dental pits
D. Odontogenic cysts
E. Natal teeth

7
Correct answer: (B) Retention of primary teeth.
Explanation: Retention of primary teeth is seen in Hyper IgE Syndrome.
(A) Pegged teeth are seen in anhidrotic ectodermal dysplasia, incontinentiapigmenti, and congenita syphilis.
(C) Dental pits are seen in Tuberous Sclerosis.
(D) Odontogenic cysts are seen in Gorlin Syndrome and Gardner Syndrome.
(E) Natal teeth are seen in Pachyonychiacongenita type II (Jackson-Sertole).

16. A patient with the following clinical findings and a history of intussusception should be screened for which of
the following genetic mutations?

A. STK11
B. PTEN
C. PTPN11
D. PRKAR1A
E. BRAF

Correct answer: (A) STK11.


Explanation: Oral melanotic macules with a history of intussusception should raise clinical suspicion for Peutz-
Jeghers Syndrome. In this syndrome, patients are born with numerous oral, genital, and acral pigmented macules,
most of which resolve by adolescence, however mucosal lesions persist. The intussusception is secondary from
numerous and variably sized hamartomatous polyps growing into the lumen of the gastrointestinal system. The
mutation associated with Peutz-Jeghers is the STK11 gene that codes for a serine threonine kinase. These patients
are also at risk for gastrointestinal adenocarcinomas, ovarian, and breast carcinomas, but can have a normal
lifespan if their cancers are detected early enough.
(B) People with mutations in the PTEN gene have a condition called PTEN Hamartoma Tumor Syndrome, or PHTS.
In the past, people with PTEN mutations were sometimes given diagnoses of Cowden Syndrome (CS), Bannayan-
Riley-Ruvalcaba Syndrome (BRRS), PTEN-related Proteus Syndrome or Proteus-like Syndrome, depending upon
their clinical presentation.However, Peutz-Jeghers syndrome is associated with GI hamartomas and
intussusception.
(C) PTPN11 is mutated in LEOPARD syndrome, which stands for lentigines, EKG abnormalities, ocular
hypertelorism, pulmonic stenosis, abnormal genitalia, retardation of growth, and sensorineural deafness.
Melanotic macules are a feature of this genetic condition and can affect the entire cutaneous surface, but tend to
spare the mucous membranes.
(D) PRKAR1A is mutated in the Carney Complex, or NAME/LAMB syndrome. This acronym details the clinical
features seen in these patients, Nevi, Atrial Myxomas, Ephelides, Lentigines, Epithelioid Blue Nevi and
MelanoticSchwannomas. Pigmented lesions in this condition can involve the vermillion border of the lips, but
gastrointestinal polyposis and its related intussusception is not a feature of the Carney Complex.
(E) BRAF is a gene mutation seen in the melanomas and is the target of vemurafenib.

8
17. This child was referred by his dentist for the the dental findings below. Upon further examination, he was also
noted with sparse hair and heat intolerance. The palate was intact. These findings are consistent with which
disorder?

A. Clouston syndrome
(hidrotic ectodermal dysplasia)
B. Christ-Siemens-Touraine syndrome
(anhidrotic ectodermal dysplasia)
C. Hay-Wells syndrome (ankyloblepharon-
ectodermal dysplasia-clefting syndrome)
D. Jackson Lawler (pachyonychiacongenita
type II)

Correct answer: (B) Christ-Siemens-Touraine syndrome (anhidrotic ectodermal dysplasia).


Explanation: Christ-Siemens-Touraine syndrome (anhidrotic ectodermal dysplasia) presents with peg-shaped teeth,
anhidrosis, and alopecia.
(A) Clouston syndrome (hidrotic ectodermal dysplasia) is an autosomal dominant disorder characterized by sparse
scalp and eyebrow hair, palmoplantar hyperkeratosis, and nail dystrophy. There are no teeth nor sweat
abnormalities associated with this condition.
(C) Hay-Wells syndrome (ankyloblepharon-ectodermal dysplasia-clefting syndrome) has a major symptom of cleft
palate/cleft lip.
(D) Jackson Lawler (pachyonychiacongenita type II) is a distractor that is not associated with pegged teeth,
anhidrosis, nor alopecia.

18. Which of the following proteins is a dimer?


A. Intermediate filaments
B. Collagen IV
C. Collagen I
D. Gap Junctions
E. Elastic Fibers

Correct answer: (A) Intermediate filaments.


Explanation: Keratin filaments, or intermediate filaments, are heterodimers composed of one acidic type and one
basic type of keratin.
(B) (C) All Collagen types are formed by the polymerization of 3 alpha-helical chains.
(D) Gap Junctions are formed when 6connexins form a connexon.
(E) Elastic Fibers are formed by an elastin core that is surrounded by numerous microfibrils.

9
19. Histopathologic exam of the lesions illustrated below is most consistent with which of the following
descriptions?
A. Normal epidermis, with a dermis populated
by heavily pigmented bipolar dendritic
spindled cells in a sclerotic appearing stroma
B. Numerous nests of melanocytes located
at the tips of rete ridges
C. Elongation of rete ridges with an increased
number of basal melanocytes
D. Mucosal epithelia with increased melanin
pigmentation in the basal layer, more
prominent at the tips of the rete ridges
E. Mucosal epithelia with increased melanin
pigmentation in the basal layer with no
distinct prominence of the rete ridges

Correct answer: (D) Mucosal epithelia with increased melanin pigmentation in the basal layer, more prominent
at the tips of the rete ridges.
Explanation: Mucosal melanotic macules, as illustrated above, demonstrate melanocytic hyperplasia with
increased melanin production at the basal layer. This is more prominent in rete ridges but can be seen throughout
the mucosal eptithelia where the melanotic macules are present.
(A) Details the major histopathologic features of a common blue nevus. Additionally, common blue nevi do not
demonstrate mitotic figures, and the melanocytes have a tendency to aggregate around appendages, blood
vessels and nerves.
(B) Describes the typical findings in junctional melanocytic nevi.
(C) Describes the histopathologic findings in lentigines. In contrast to post-inflammatory hypermelanosis, there is
no increased melanin deposition in the dermis, nor are there increased melanophages.
(E) Describes the histopathologic finding of post-inflammatory hypermelanosis. These lesions histopathologically
demonstrate a greater abundance of melanin deposition in the lamina propria with increased numbers of
melanophages with a more prominent lymphocytic infiltrate when compared to lesions of oral melanotic macules.

20. A patient presents with multiple lentigines, Leydig cell tumor, and Cushing syndrome.
Which test would best aid in identifying the associated syndrome?
A. MRI head
B. CT Abdomen
C. Chest X-Ray
D. Echocardiogram
E. Colonoscopy

Correct Answer: D. Echocardiogram.


Explanation: Carney Complex, also known as LAMB or NAME syndrome is characterized by autosomal dominant
inheritance, mutations in PRKAR1A gene (tumor suppressor gene). LAMB stands for Lentigines, Atrial Myxomas,
and Blue nevi. NAME syndrome stands for Nevi, Atrial myxomas, Myxoidneurofibromas, and Ephelides. In this
presentation, the next test to order would be an echocardiogram to assess for atrial myxoma.
Incorrect Answers:
A. MRI head is not indicated if suspicious for Carney Complex.
B. CT abdomen is not indicated if suspicious for Carney Complex.

10
C. Chest x-ray is not indicated if suspicious for Carney Complex.
E. Colonoscopy is not indicated if suspicious for Carney Complex.
Practice Connection: Other features include pigmented adrenocortical disease,
psammomatousmelanoticschwannomas, pituitary adenoma and testicular tumors.

21. Which of the following associations of Ataxia-Telangiectasia is the most common cause of death?
A. Respiratory Failure
B. Lymphoma
C. Leukemia
D. Breast Cancer
E. Staph Aureus infections

Correct answer: (A) Respiratory Failure.


Explanation: Patients with Ataxia-Telangiectasia (AT) have several complications due to a primary B and T-cell
immunodeficiency. Recurrent sinopulmonary infections are seen in more than 80% of AT patients and lead to the
destruction and widening of the large airways. Bronchiectasis, resulting from this destruction, with respiratory
failure is the most common cause of death in AT patients.
(B) (C) While there is an increased risk of Leukemia (70 fold) and Lymphoma (200 fold) when compared to the
unaffected individuals, these are not the most common causes of death in AT.
(D) Heterozygotes that carry the ATM mutation are at an increased risk of Breast Cancer, as well as death due to
solid organ carcinoma.
(E) The immunodeficiency associated with ATM predisposes its patients to superficial pyodermas and abscesses,
commonly caused by Staph Aureus, however these infectious manifestations are not the most common cause of
death in AT.

22. What lab abnormality would you most likely expect in a patient with Blue Rubber Bleb Nevus Syndrome?
A. Thrombocytopenia
B. Thrombocytosis
C. Leukopenia
D. Anemia
E. Neutropenia

Correct answer: (D) Anemia.


Explanation: Anemia is the most likely lab abnormality to be found in patients with Blue Rubber Bleb Nevus
Syndrome secondary to venous malformations/hemorrhage in the gastrointestinal tract. In addition, patients
present with multiple venous malformations on the trunk and extremities.

23. Cornelia de Lange syndrome is not characterized by which of the following?


A. Lymphedema
B. Cutis marmorata
C. Trichomegaly
D. Intellectual disability
E. Low-pitched cry

Correct answer: (A) Lymphedema.


Explanation: Cornelia de Lange syndrome is characterized by autosomal dominant or sporadic inheritance, NIPBL
(nipped-beta-like gene) mutations, cutis marmorata, hirsutism, trichomegaly, craniofacial abnormalities,
intellectual disability, deafness, low-pitched cry, synophrys, short stature, and clinodactyly.

11
24. Identify the gene product that is deficient in the genodermatosis that is described: A patient with a history of mild
erythroderma and generalized peeling in his neonatal period. His mother told him that a cesarian section had to be
performed as labor failed to progress. Later in life, he developed the typical polygonal, dark-brown, adherent
scales, distributed symmetrically on the extremities, trunk and neck. His neck is involved and can be described as a
« dirty neck ». Other extensor surfaces of his body are affected. His palms, soles and face are spared.
A. Transglutaminase-1
B. Filaggrin
C. ATB binding cassette 12
D. Steroid sulfatase
E. Serine protease inhibitor LEKTI

Correct answer: (D) Steroid sulfatase.


Explanation: To correctly answer this question, a good knowledge of the key features in clinical presentation of the
ichtyoses is required. The two main clues are the delayed labor and the distribution of the lesions. X-linked
ichthyosisis caused by a mutation affecting steroid sulfatase. The low or absent levels of estrogen in the urine and
amniotic fluid is caused by the inadequate deconjugation of DHEAS, a necessary step for estrogen synthesis. As a
result, labor often fails to initiate spontaneously or to progress. This is due to insufficient dilation of the cervix and
can only be partially overcome by oxytocin administration. In many cases, a cesarean section is required. The
flexural areas (including the popliteal and antecubital fossae, palms and soles, and the central face) are often
spared, but may be involved.
Ichthyosis vulgaris is caused by a mutation in filaggrin. Lamellar ichtyosisis caused by a transglutaminase-1
mutation. It can present with a collodion baby. Ectropion and eclabium are generally part of the clinical picture as
well as palmoplantarkeratoderma.Clinically, ichthyosis vulgaris is distinguished from X-linked ichthyosis by an
association with hyperlinear palms and keratosis pilaris.Individuals with Harlequin ichthyosis are usually born
prematurely at a mean gestational age of 35 weeks and often die within a few days or weeks because of
respiratory insufficiency or sepsis. The disorder is due to a mutation in the ATB binding cassette 12.
Finally, a mutation in SPINK5 results in a deficient serine protease inhibitor LEKTI in Comèl-Netherton syndrome.
The lesions evolves into serpiginous or circinate scaling and erythematous plaques bordered by a peculiar double-
edged scale and descriptively named “ichthyosislineariscircumflexa”. The plaques are usually distributed on the
trunk and extremities and change over time in size, shape and location. Hair shaft abnormalities usually develop
during infancy and early childhood and improve with age. Trichorrhexisinvaginata (bamboo hair) with a ball-and-
socket appearance is found. Another feature of the syndrome is immune imbalance. Serum levels of IgEare usually
markedly elevated. Eosinophilia and allergic reactions to various foods (e.g. nuts, eggs, fish) and other antigens
are common.

25. In patients with incontinentiapigmenti (Bloch-Sulzberger syndrome), at what age do verrucous lesions typically
appear?
A. Birth to 2 weeks
B. 2-6 weeks
C. 3 to 6 months
D. 20 years
E. 40 years

Correct answer: (B) 2-6 weeks.


Explanation: IncontinentiaPigmenti is X-linked dominant and secondary to a mutation in the NEMO gene (nuclear
factor NF-kappa beta essential modulator). There are 4 stages of cutaneous lesions: 1) Vesicular: from birth to 2

12
weeks, 2) Verrucous: from 2-6 weeks, 3 )Hyperpigmentation: from 3-6months, and 4) Hypopigmenation: in the
second to third decade. Other key features include peg-shaped teeth, alopecia, and CNS and eye abnormalities.

26. A patient of Puerto Rican heritage presents to clinic with cream colored hair, photophobia, and frequent
nosebleeds. On physical exam, the patient is noted to have multiple petechiae and pigmented nevi. What
syndrome does this patient have?
A. Oculocutaneous Albinism Type II
B. Griscelli Syndrome
C. Chediak-Higashi Syndrome
D. Piebaldism
E. Hermansky-Pudlak Syndrome

Correct answer: (E) Hermansky-Pudlak Syndrome.


Explanation: Hermansky-Pudlak Syndrome is characterized by autosomal recessive inheritance, HPS gene mutation
(lysosomal transport protein), pigment dilution, pigmented nevi, increased squamous/basal cell cancer,
photophobia, prolonged bleeding, and ceroid (chromolipid) deposition in macrophages in the lung (pulmonary
fibrosis), gastrointestinal tract (granulomatous colitis) and heart (cardiomyopathy).
(A) Oculocutaneous Albinism Type II is characterized by autosomal recessive inheritance, mutation in P gene with
decreased eumelanin synthesis, generalized pink-white/cream color, blue to yellow-brown irides, nystagmus,
photophobia, pigmented nevi, and increased squamous/basal cell carcinoma.
(B) Griscelli Syndrome is characterized by autosomal recessive inheritance, mutations in Rab 27A and MyO5A
(myosin V), silvery hair, partial albinism, pancytopenia with increased infections, progressive neurologic
deterioration.
(C) Chediak-Higashi Syndrome is characterized by autosomal recessive inheritance, LYST gene mutation, light
blond/silvery hair, oculocutaneous albinism, ataxia, giant lysosomal granules, recurrent infections, photophobia,
ataxia, muscle weakness, and other hematologic abnormalities.
(D) Piebaldismis characterized by autosomal dominant inheritance, mutation in c-kit proto-oncogene,
depigmented patches with islands of hyperpigmentation and white forelock.

27. This infant was noted at birth with the skin findings below. Characteristic features of this condition include all
of the following, except?

A. Histologic evidence of loss or fragmentation


of elastic fibers
B. Fibulin-5 gene defects
C. Gastrointestinal diverticula
D. Arterial aneurysms
E. None of the above. All are true.

13
Correct answer: (E) None of the above. All are true.
Explanation: All of the statements are correct.
(A) Cutis laxa presents with loose, redundant sagging skin that appears at birth with progression throughout life.
Histologically, loss and fragmentation of elastin fibers are evident.
(B) Fibulin 5 gene defects are associated with autosomal recessive cases of cutis laxa.
(C) GI and GU diverticula are associated with cutis laxa.
(D) Vasular anomalies and arterial aneurysms are associated with cutis laxa.

28. An ATP-binding cassette transporter C-6 defect can lead to which of the following diseases?
A. Menkes disease
B. Hailey-Hailey disease
C. Pseudoxanthomaelasticum
D. Harlequin ichthyosis
E. Hutchinson-Gilford progeria syndrome

Correct answer: (C) Pseudoxanthomaelasticum.


Explanation: An ATP-binding cassette transporter C-6 (ABCC6) defect can lead to pseudoxanthomaelasticum.
Incorrect answers:
(A) Menkes disease is due to a mutation in the gene encoding the ATP7A copper transporter.
(B) Hailey-Hailey disease is caused by a calcium pump defect related to a mutation in the ATP2C1 gene on
chromosome 3, which encodes a Ca2+ ATPase localized to the Golgi apparatus.
(D) Harlequin ichthyosis is also caused by an ATP-binding cassette transporter, but this time, by ABCA12, but not
ABCA6.
(E) Hutchinson-Gilford progeria syndrome is caused by a mutation in LMNA, a nuclear envelopathy.

29. All of the following are extracutaneous associations of Sturge-Weber Syndrome EXCEPT?
A. Growth Hormone Deficiency
B. Hypothyroidism
C. Attention Deficit Disorder
D. Transient Ischemic Attacks
E. All of the above are features of Sturge-Weber Syndrome

Correct answer: (E) All of the above are features of Sturge-Weber Syndrome.
Explanation: Struge-Weber syndrome is a congenital vascular disorder characterized by a port wine stain in the
distribution of the first or second trigeminal nerve. The most frequent extracutaneous manifestations of Sturge-
Weber Syndrome (SWS) are those that affect the ophthalmologic and neurologic systems. Leptomeningeal
involvement can lead to intracranial vascular malformations and present with seizures, headaches, cognitive
impairment, attention deficit disorder, transient ischemic attacks and stroke. Ophthalmologic evaluations should
be regular, and is also recommended in patients without SWS but with V1 capillary malformations. The most
frequent ocular complication is glaucoma with the added potential of visual loss, but increased ocular pressure and
choroidal vascular anomalies are also part of the spectrum of eye involvement in SWS. Finally, increased reports of
associated growth hormone deficiency and hypothyroidism in patients with SWS also warrant frequent assessment
for these complications.

14
30. A 4 y/o female is brought to your clinic for a discussion of management options following the enlargement of a
violaceous tumor growing on the posterior and lateral aspect of her tongue. She was recently diagnosed with
obstructive sleep apnea that has only worsened with the tumors increase in size. It is not painful, is of normal
mucosal temperature, and is without a pulse. Which of the following treatment options is considered first-line
for this lesion?
A. Excision
B. Sclerotherapy
C. Endovenous Laser Ablation
D. Topical Imiquimod
E. Pulsed-Dye Laser

Correct answer: (B) Sclerotherapy.


Explanation: Sclerotherapy is the first-line treatment for venous malformations. This vignette details a venous
malformation of the oral mucosa. While initially these lesions can be asymptomatic and subtle in their
appearance, over time they can grow in size leading to additional complications based on location. Venous
malformations of the oral cavity predispose patients to bony malformations of the temporal, orbital, and maxillary
bone, while those venous malformations at the posterior aspect of the oropharynx can lead to obstructive sleep
apnea. The decision to treat a venous malformation greatly depends on the potential for disfigurement and/or
functional impairment. Sclerotherapy consists of the introduction of caustic materials into the veins in order to
induce damage to the endothelial cells lining the internal lumen of the veins. Damage to the endothelial cells leads
to scarring of the vascular lumen and eventually collapse of the vein. Without a continued vascular supply, the
veins eventually reabsorb into local tissue.
(A) (C) These are valid therapeutic options for venous malformations, however, surgical excision is often reserved
for smaller localized lesions, or larger lesions in combination with sclerotherapy. The location of the venous
malformation in this vignette would be more amenable to sclerotherapy than an excision given its location in the
oral cavity. Radiofrequency and laser ablation therapy are more invasive techniques that are utilized in the
treatment of venous malformations, but these modalities are not considered first-line therapies.
(D) (E) These are potential treatments for capillary malformations and not venous malformations.

31. The lesions depicted below can be associated with which of the following?

A. Rombo syndrome
B. Brooke-Spiegler syndrome
C. Bazex-Dupre-Christol syndrome
D. A&B
E. All of the above

15
Correct answer: (E) All of the above.
Explanation: All of the above syndromes can present with trichoepitheliomas.
(A) Trichoepitheliomas are benign tumors that present as discrete papules along the nasolabial folds. Rombo
syndrome presents with trichoepitheliomas in addition to milia, hypotrichosis, basal cell carcinomas and
atrophoderma.
(B) Brooke-Spiegler syndrome presents with multiple trichoepitheliomas and cylindromas.
(C) Bazex-Dupre-Christol syndrome is an X-linked dominant disorder that presents with follicular atrophoderma,
basal cell carcinomas, trichoepitheliomas, hypotrichosis and hypohidrosis.

32. A patient with a history of colon carcinoma presents to you with numerous sebaceous adenomas and a likely
sebaceous adenocarcinoma. What is the altered gene(s) in this syndrome?
A. MLH1 & MSH2
B. PTEN
C. FALDH (Fatty aldehyde dehydrogenase)
D. PAHX/PEX7
E. GJB3

Correct answer: (A) MLH1 & MSH2.


Explanation: MLH1 & MSH2 genes are involved in DNA mismatch repair and are altered in Muir-Torre Syndrome,
which is characterized by autosomal dominant inheritance, multiple sebaceous tumors (adenomas, carcinomas,
hyperplasias), basal cell carcinomas with sebaceous differentiation, keratoacanthomas, and gastrointestintal tract
malignancy.
(B) PTEN is involved in tumor suppression and is mutated in Cowden Syndrome, which is characterized by
autosomal dominant inheritance, trichilemmomas, oral papillomas, cobblestoning of the oral mucosa, mostly
benign hamartomatous GI polyps, increased breast/thyroid/colon cancer and punctate keratoses of the palms and
soles.
(C) FALDH (Fatty aldehyde dehydrogenase) is altered in Sjogren-Larsson Syndrome. Key characteristics include
autosomal recessive inheritance, ichthyosis, pruritus, intellectual disability, spastic di or triplegia, scissor gait,
epilepsy, and atypical retinal pigment degeneration in macula (atypical retinitis pigmentosa with “glistening dots”
pattern).
(D) PAHX/PEX7, involved respectively in phytanic acid metabolism and peroxisomal enzyme receptors, can be
altered in Refsum Syndrome, which is characterized by autosomal recessive inheritance, mild ichthyosis, ataxia,
peripheral neuropathy, retinitispigmentosa with salt and pepper pigment, cardiac arrhythmias, symmetric muscle
wasting and sensorineural deafness.
(E) GJB3 is involved in encoding Connexin 31, which is mutated in ErythrokeratodermaVariabilis, also known as
Mendes da Costa syndrome. Mendes da Costa syndrome is characterized by autosomal dominant inheritance, well
demarcated geographic erythematous patches that may change daily on face, buttocks, extremities, and fixed
hyperkeratotic plaques especially on palms and soles.

33. Which of the following is an X-linked dominant genodermatosis?


A. Chronic Granulomatous Disease
B. Menkes Disease
C. Fabry Disease
D. Type V Ehlers-Danlos
E. Goltz Syndrome

16
Correct answer: (E) Goltz Syndrome.
Explanation: Goltz Syndrome is characterized by X-linked dominant inheritance. The other answer choices are X-
linked recessive.

34. Epidermolytic Hyperkeratosis (Epidermal Ichthyosis or Bullous Congenital IchthyosiformErythroderma) is a


congenital ichthyosis due to mutations in the keratins expressed in the epidermis. Which of the following
PalmoplantarKeratoderma’s (PPK) shares a similar mutation to this hereditary ichthyosis?
A. Vohlwinkel Syndrome
B. Vorner-Type PalmoplanatarKeratoderma
C. Carvajal Syndrome
D. Mal De Meleda
E. Pappillon-Lefèvre Syndrome

Correct answer: (B) Vorner-Type PalmoplanatarKeratoderma.


Explanation: Vorner Type PalmoplantarKeratoderma, an epidermolytic PPK, as well as a single reported family
with Unna-Thost PPK, a non-epidermolytic PPK, result from genetic mutations in the genes coding Keratin 1.
EpidermolyticIchthyosis (EI) is a congenital ichthyosis due to mutations in both Keratins 1 and 10. Interestingly,
both syndromes, Vorner-type PPK and EI demonstrate epidermolysis and hyperkeratosis on pathology.
Additionally, patients with EI caused by K1 mutations also present with severe PPK, while those EI patients with a
Keratin 10 mutation do not develop PPK. It is thought that Keratin 1 and 9 form a heterodimer and are located in
the suprabasal layers of palmar and plantar skin. Unna-Thost PPK in one reported Scandinavian family was found
to have a similar mutation in Keratin 1.
(A) Vohlwinkel Syndrome is a PPK with additional clinical features such as deafness, star-shaped hyperkeratotic
plaques, and pseudoainhum and is due to mutations in either Connexin 26 (GJB2) or loricrin. The Vohlwinkel
syndrome variant that is due to mutations in loricrin, does not present with deafness, but instead a generalized
ichthyosis.
(C) Carvajal syndrome is an example of a striate PPK that presents with characteristic wooly hair. It is due to a
mutation in the desmosomal protein desmoplakin.
(D) Mal de Meleda associated PPK is a diffuse Palmoplantarkeratoderma characterized by an erythematous border
surrounding the keratodermatous palms and soles, with transgrediens spread and foul odor. It is due to a
mutation in the SLURP-1 gene.
(E) Pappillon-Lefèvre Syndrome, due to a mutation in Cathepsin C, is a diffuse transgredient PPK associated with
destructive periodontitis that begins in at an early age and results in premature loss of teeth.

35. Which of the following is characterized by elevated red blood cell (RBC) porphyrins?
A. ErythropoieticProtoporphyria (EPP)
B. Porphyria CutaneaTarda (PCT)
C. Hereditary Coproporphyria
D. Acute Intermittent Porphyria (AIP)
E. Variegate Porphyria (VP)

Correct answer: (A) ErythropoieticProtoporphyria (EPP).


Explanation: ErythropoieticProtoporphyria (EPP) has elevated porphyrins in RBCs. The three porphyrias with
elevated RBC protoporphyrins are ErythropoieticProtoporphyria (EPP), Congenital Erythropoietic Porphyria (CEP),
and Hepatoerythropoietic Porphyria (HEP). Notice that all three tyes of porphyria with elevated RBC porphyrins
contain the word "erythropoietic".
(B) Porphyria CutaneaTarda (PCT) has normal porphyrins in RBCs.
(C) Hereditary Coproporphyria is incorrect.

17
(D) Acute Intermittent Porphyria (AIP) has normal porphyrins in RBCs.
(E) Variegate Porphyria (VP) has normal porphyrins in RBCs.

36. First line therapy for a patient who was referred to you with recurrent episodes of erysipelas-like lesions on the
lower extremities accompanied by abdominal pain consists of which of the following medications?
A. Methotrexate
B. Etanercept
C. Cephalexin
D. Doxycycline
E. Colchicine

Correct answer: (E) Colchicine.


Explanation: The vignette details a patient who has a diagnosis of Familial Mediterranean Fever or FMF. This
hereditary Autoinflammatory disorder is due to mutations in the MEFV gene that encodes the protein pyrin. It is
characterized by acute attacks of high fever, serositis (peritonitis, synovitis, pleuritis, joint pain, and abdominal
pain), as well as an erysipelas- or cellulitis-like eruption of the lower extremity. Similar lesions are seen in the
pediatric population without the accompaniment of fevers. First line treatment is Colchicine (1-2.5mg/kg per day)
which not only prevents recurrence of the disease, but also prevents the long term complication of systemic
amyloidosis.
(A) (B) These are immunomodulators that act on suppressing the immune system in auto-immune inflammatory
diseases such as psoriasis, or in the case of Methotrexate, recalcitrant inflammatory diseases such as atopic
dermatitis.
(C) (D) These are both antibiotics, and while useful as therapeutic modalities in the treatment of bacterial
infections, are not part of the therapeutic ladder in Autoinflammatory diseases such as FMF.

37. Which of the following statements regarding Familial Cold Urticaria is correct?
A. Responsive to anti-histamines
B. It is possible to elicit a reaction by an ice-cube test
C. The predominant finding on histopathology is an eosinophillic infiltrate
D. Significant hypothermia develops after exposure to cold
E. Patients must be subjected to cold environmental temperatures in order to elicit a response

Correct answer: (E) Patients must be subjected to cold environmental temperatures in order to elicit a
response.
Explanation: To elicit a response in a patient suspected of having Familial Cold Urticaria patients require exposure
to cold temperatures or cold-water immersion.
(A) To further contrast acquired cold urticaria, responses to anti-histamines, both H1 and H2 blockers, are
relatively poor in Familial Cold Urticaria.
(B) The application of an ice-cube to the skin would not induce urticarial lesions. The application of an ice-cube to
the skin with the immediate development of wheals is often seen in acquired cold urticaria. Patients with Familial
Cold Urticaria do not develop cutaneous lesions immediately. Cutaneous and extracutaneous reactions are often
delayed up to 6 hours following a cold challenge.
(C) The predominant finding on histopathology of a biopsy from a cutaneous wheal in a patient with Familial Cold
Urticaria, demonstrates a neutrophillic, and not eosinophillic infiltrate.
(D) Finally, contrary to the name of this disorder, following exposure to the cold, and especially with moist and
windy weather, patients tend to develop fever and not hypothermia.

18
38. What type of Ehlers-Danlos Syndrome is characterized by increased risk of arterial and visceral rupture?
A. Type 1
B. Type 2
C. Type 3
D. Type 4
E. Type 6

Correct answer: (D) Type 4.


Explanation: Type 4 Ehlers-Danlos is known as vascular type and is characterized by autosomal dominant
inheritance, defect in COL3A1(Type III Collagen), translucent skin with visible veins, and increased arterial and
visceral rupture usually leading to early death.
(A) Type 1 Ehlers-Danlos is known as the classic type and characterized by autosomal dominant inheritance, defect
in COL5A1 or COL5A2 (Type 5 Collagen), hyperextensible skin, joint laxity, gaping wounds, cigarette-paper scars,
calcified subcutaneous nodules, bruising, absence of frenulum, positive Gorlin sign, molluscoidpseudotumors,
mitral valve prolapse, and blue sclerae.
(B) Type 2 Ehlers-Danlosis known as Mitis type, is characterized by autosomal recessive inheritance, Tenascin X
Deficiency, and is a milder form of Type I.
(C) Type 3 Ehlers-Danlos is known as the hypermobility type, is characterized by autosomal dominant inheritance,
defect in in TNXB (Tenascin X), severe joint hyperextensibility, recurrent dislocations, and minimal skin
involvement.
(E) Type 6 Ehlers-Danlosis known as kyphoscoliosis type and is characterized by autosomal recessive inheritance,
defect in PLOD (Lysyl hydroxylase), kyphoscoliosis, muscle weakness, and ocular fragility.

39. Which of the following components of the workup for patients with Neurofibromatosis Type 1 is essential?
A. Brain and Spinal MRI
B. Ophthalmologic Exam
C. Renal Arteriography
D. Yearly Audiograms
E. Echocardiogram

Correct answer: (B) Ophthalmologic Exam.


Explanation: The workup and management of patients suspect of having, and then diagnosed with NF-1, requires a
multidisciplinary approach often involving Dermatologists, Neurologists, Ophthalmologists, and General
Practitioners. While plexiformneurofibromas, and their potential to degenerate into malignant peripheral nerve
sheathe tumors is of significant concern for dermatologists throughout the lifetime of patients with NF-1,
ophthalmologic exam with visual assessment is the next most essential evaluation to perform. The risk of optic
gliomas which can lead to eventual blindness are found in up to 15% of patients with NF-1. Their symptoms range
from non-existent to proptosis with increased tumor size and complete visual compromise. It is therefore
recommended that patients with NF-1 receive and ophthalmologic exam yearly until the age of 8. More recent
literature suggests that it continue until the age of 10. The remaining choices, aside from choice (D), are
recommendations based on potential complications and associations with NF-1. Brain and Spinal MRI should be
reserved if neurological signs and symptoms develop or following the diagnosis of optic glioma following
ophthalmologic exam. Renal arteriography, along with 24-hour urine catecholamines and metanephrinesshould be
reserved unless hypertension develops as 1% of NF-1 patients will develop pheochromocytoma. Essential
hypertension is the leading cause of HTN in NF-1 patients but renal stenosis and pheochromocytoma are potential
contributors and should be ruled out. Echocardiograms should be conducted only after auscultation of a murmur is
detected.(D) Yearly audiograms is part of the work-up and management for Neurofibromatosis Type 2.

19
40. A woman has a genetic mutation on one chromosome for focal dermal hypoplasia and asks her doctor about the
likelihood of her unborn daughter developing the same condition.
A. Lethal in females
B. 25%
C. 50%
D. 75%
E. 100%

Correct answer: (C) 50%.


Explanation: Focal Dermal Hypoplasia, also known as Goltz syndrome, is inherited in an X-linked Dominant
fashion. It is lethal in male hemizygotes and the chance of female inheritance is 50%. Key features include PORCN
gene mutation, asymmetric atrophic hyper- or hypopigmentedtelangiectatic linear streaks in Blaschko’s lines on
trunk and extremities, alopecia, fat herniation, osteopathiastriata, dystrophic nails, enamel pits, and
mucocutaneouspapillomas.

41. All of the following are examples of Diffuse PalmoplantarKeratoderma EXCEPT?


A. Carvajal Syndrome
B. Naxos Disease
C. Unna-Thost
D. Vohlwinkel Syndrome
E. Pappillon-Lefèvre Syndrome

Correct answer: (A) Carvajal Syndrome.


Explanation: Carvajal Syndrome is classified as a focal PalmoplantarKeratoderma (PPK). It is further categorized as
a striate form of focal PPK. The remaining choices (B-E) are all considered diffuse PPK’s. Several attempts to
classify the PPK’s have yielded little consensus regarding the various categories to which each individual entity
could fall under. One commonly referred to strategy was to divide them into three broad categories (diffuse, focal,
and punctate), with further stratification based on the presence of absence of associate features. Diffuse PPK’s
involve the entire surface of the palms and soles. Several PPK’s in this category may demonstrate transgrediens
involvement, or involvement of the keratoderma to the lateral and dorsal aspects of the hands, feet, and flexor
aspects of the wrists and heels. Focal PPK’s can be further classified into areata/nummular or striate types, and are
characterized by involvement mainly over the pressure points, or in a linear pattern respectively. Carvajal
syndrome is most consistent with a focal PalmoplantarKeratoderma of the linear type. Punctate PPK’s are
characterized by the presence of 1mm to 1 cm hyperkeratotic papules on the palms and soles.

42. Select the pattern of inheritance that best illustrates the following scenario: The mother of an affected child has
a mosaic pattern of skin lesions. One of her two daughters show the same skin lesions in the same distribution.
She had two spontaneous abortions of male fetuses and was told the abortions were related to her condition.
A. X-linked recessive
B. Autosomal dominant
C. Autosomal recessive
D. X-linked dominant
E. Y-linked

Correct answer: (D) X-linked dominant.


Explanation: The patterns of inheritance are described below: Pattern: X-linked dominant Parents affected: Yes
Gender affected: Mostly females if lethal in males during embryonic development. Otherwise milder in females
than in males. Often with a mosaic pattern of skin lesions in females. Transmission: Affected males have: no

20
affected sons and all daughters affected. No male-to-male transmission. Recurrence risk: 1 in 2 children born to
affected female. May spontaneously abort male fetuses if “male lethal” condition. Risk factors: De novo
mutations; tend to be lethal in males.
*** Pattern: X-linked recessive. Parents affected: The mother is a carrier or is mildly affected. Gender affected:
Males have the full-blown disease. Female may have mild manifestations in a mosaic pattern. Transmission: No
male-to-male transmission. All daughters of an affected male are carriers. Recurrence risk: 1 in 2 male children
born to a female carrier will be affected. 1 in 2 of her female children will be carriers. Risk factors: De novo
mutations.
*** Pattern: Autosomal recessive. Parents affected: No, the parents are carriers, but do not harbors symptoms or
signs. Gender affected: Both are affected equally. Transmission: The disease manifests in siblings. It does not
manifest in parents or offspring of the affected patient. It is usually seen in one generation. Recurrence risk: 1 in 4
pregnancies with no effect of the gender. Risk factors: Consanguinity and isolated populations.
*** Pattern: Autosomal dominant. Parents affected: Yes (one of them). Gender affected: Both equally (male and
female). Transmission: The disease is seen in every generation. Recurrence risk: 1 in 2 pregnancies with no effect
of the gender. Risk factors: De novo mutations.
*** Pattern: Y-linked (or holandric inheritance) Parents affected: Father Gender affected: Only males Transmission:
Father of boy always affected. Recurrence risk: All sons of affected father are affected. Risk factors: There are
relatively few Y-linked disorders. Nice to know: Most Y-linked disorders affect male fertility.

43. What is the most common nevus in in patients with Carney Complex?
A. Spitz Nevus
B. Epidermal Nevus
C. Blue Nevus
D. Becker’s Nevus
E. Nevus Sebaceous

Correct answer: (C) Blue Nevus.


Explanation: Carney Complex, also known as LAMB or NAME syndrome is characterized by autosomal dominant
inheritance, mutations in PRKAR1A gene (tumor suppressor gene). LAMB stands for Lentigines, Atrial Myxomas,
and Blue nevi. NAME syndrome stands for Nevi, Atrial myxomas, Myxoidneurofibromas, and Ephelides. Other
features include pigmented adrenocortical disease, psammomatousmelanoticschwannomas, pituitary adenoma
and testicular tumors.

44. Which of the following tumors of neural origin is the most common cutaneous finding in Neurofibromatosis Type 2?
A. Cutaneous Neurofibromas
B. Café Au Lait Macules
C. MyxoidNeurothekeomas
D. Neurilemomas
E. Melanocytic Nevi

Correct answer: (D) Neurilemomas.


Explanation: Neurilemomas, or Schwannomas, are the most common cutaneous manifestation in
Neurofibromatosis Type 2 (NF-2) and can be found in up to 66% of patients with this disease. While bilateral
acoustic schwannomas are pathognomonic for NF-2, they present with conductive hearing loss and not as a
cutaneous finding. (B) While CALMs may be present in patients with NF-2, up to 50%, they are far fewer in number
when compared to NF-1, and are almost always fewer then 6. Additionally, when compared to NF-1, patients do
not have Lisch Nodules or learning deficits such as intellectual disability or attention deficit disorder, both of which
are often seen in NF-1. (A) (C) (E) These are incorrect.

21
45. Which of the following statements regarding Hailey-Hailey Disease is INCORRECT?
A. The primary lesions are easily rupture vesicles that give rise to macerated plaques
B. Hypertrophic scarring and Keloid formation is common at sites of resolved Hailey-Hailey
Disease
C. Fetid odor and pruritus have a significant impact on psychosocial morbidity
D. Heat, friction, secondary infection, and sweat are common causes of disease exacerbation
E. Kaposi’s Varicelliform eruption can complicate patients with Hailey-Hailey Disease

Correct answer: (B) Hypertrophic scarring and Keloid formation is common at sites of resolved Hailey-Hailey
Disease.
Explanation: Choice (B) is incorrect. Lesions of Hailey-Hailey Disease heal WITHOUT scar formation. Rupturing of
the vesicular lesions in Hailey-Hailey Disese leaves behind macerated and vegetative plaques, predominantly in the
inguinal, axillary, and inframammary areas of the body. Unlike Darier disease, Hailey-Hailey Disease can involve
the gluteal cleft. In areas that are not intertriginous, the lateral neck and superior shoulder, lesions of Hailey-
Hailey Disease appear less macerated and vegetative and can be mistaken for lesions of Darier disease. Like Darier
Disease, lesions of Hailey-Hailey Disease are complicated by the same exacerbating factors such as heat, occlusive
clothing, and secondary viral, dermatophyte, and bacterial infection. Superimposed HSV can cause Kaposi’s
Varicelliform eruption in patients with Hailey-Hailey Disease. Additionally, these same factors can lead to
malodorous skin lesions, which then significant impact a patient’s social function.

46. Which of the following statements regarding Tuberous Sclerosis is TRUE?


A. Cardiac rhabdomyomas associated with Tuberous Sclerosis last throughout adulthood and
warrant frequent echocardiography.
B. Risks for the development for Autism are increased in patients with Tuberous Sclerosis and
behavioral evaluation is recommended for children
C. Pulmonary Lymphangioleiomyomatosis has an equal incidence in male and female Tuberous
Sclerosis patients
D. Ocular lesions in Tuberous Sclerosis often lead to blindness by age 18
E. None of the above are true, A-D are all false statements.

Correct answer: (B) Risks for the development for Autism are increased in patients with Tuberous Sclerosis
and behavioral evaluation is recommended for children.
Explanation: In addition to autism, attention deficit disorder, and psychoses are also at an increased risk of
developing in patients with TS. Mental deficits and intellectual impairment occur with a higher frequency when
compared to the general population in patients with TS.
(A) Cardiac rhabdomyomas, while a major feature of TS, often involute within the first 3 years of life. They have a
prevalence of 80% in TS patients and may be solitary or multiple.
(C) Lymphangioleiomyomatosis, which appears as cysts on high resolution pulmonary CT, is rarely found in male
patients with TS. Estrogen is believed to play a stimulatory role in the growth of smooth muscle cells in the lungs
that lead to the growth of this tumor.
(D) Ocular lesions in TS, which are predominantly retinal hamartomas, do not lead to visual loss.

47. What eye finding is a feature of Gardner’s Syndrome?


A. Photophobia
B. Retinoblastoma
C. Corneal vascularization
D. Lid papillomas
E. Congenital hypertrophy of retinal pigment epithelium (CHRPE)

22
Correct answer: (E) Congenital hypertrophy of retinal pigment epithelium (CHRPE).
Explanation: Gardner Syndrome is characterized by autosomal dominant inheritance, mutations in APC (tumor
suppressor gene), epidermoid cysts (especially of the head and neck), fibromas, osteomas (especially of the
mandible and maxilla), gastrointestinal polyposis with high rate of malignant adenocarcinoma, desmoid tumors
(after abdominal surgery, uterine), odontomas, and congenital hypertrophy of retinal pigment epithelium (CHRPE).

48. A 2 year-old patient presenting with palmoplantarkeratoderma and wooly hair since birth likely has a mutation
in which of the following proteins?
A. Keratin 1
B. Desmoglein 1
C. Plakoglobin
D. SLURP-1
E. Connexin 26

Correct answer: (C) Plakoglobin.


Explanation: The brief vignette details the combination of palmoplantarkeratoderma (PPK) and wooly hair
representing one of two syndromes. Plakoglobinis mutated in Naxos syndrome, which presents with wooly hair
and diffuse non-epidermolytic PPK at birth, and onset of arrhythmias and RIGHT sided cardiomyopathy in
adolescence. Carvajal syndrome, similar in its presentation of a PPK and wooly hair, differs from Naxos in several
ways. The PPK associated with Carvajal Syndrome is epidermolytic, striate, non-diffuse and there is more extensive
involvement of the ectoderm. This can include nail dystrophy, hypodontia and skin fragility. What is most
important to remember when differentiating Carvajal syndrome from Naxos is that the dilated cardiomyopathy
associated with it tends to involve the LEFT side of the heart.
(A) Vorner Type PalmoplantarKeratoderma, an epidermolytic PPK, as well as a single reported family with Unna-
Thost PPK, a non-epidermolytic PPK, result from genetic mutations in the genes coding Keratin 1.
(B) Desmoglein 1 is mutated in some forms of striate PPK, but is not a mutation found in PPK’s with wooly hair.
(D) Mal de Meleda associated PPK is a diffuse Palmoplantarkeratoderma characterized by an erythematous border
surrounding the keratodermatous palms and soles, with transgrediens spread and foul odor. It is due to a
mutation in the SLURP-1 gene.
(E) Vohlwinkel Syndrome is a PPK with additional clinical features such as deafness, star-shaped hyperkeratotic
plaques, and pseudoainhum and is due to mutations in either Connexin 26 (GJB2) or LOR.

49. The patient below was referred to your clinic by a head and neck surgeon who removed a salivary gland tumor.
What syndrome is most likely present in the following patient?

A. Birt-Hogg-Dube
B. Brooke-Spiegler Syndrome
C. Neurofibromatosis Type II
D. Buschke-Ollendorf Syndrome
E. Peutz-Jeghers Syndrome

23
Correct answer: (B) Brooke-Spiegler Syndrome.
Explanation: Brooke-Spiegler Syndrome is secondary to a CYLD gene mutation and is characterized by
trichoepitheliomas over the nasolabial folds, nose, forehead, and ears, cylindromas and spiradenomas. Affected
individuals are also at increased risk of developing tumors in tissues other than skin appendages, particularly
benign or malignant tumors of the salivary glands.
(A) Birt-Hogg-Dube is characterized by autosomal dominant inheritance, BHD gene mutation, numerous
fibrofolliculomas, acrochordrons and renal tumors (papillary carcinoma, oncocytomas, chromophobe adenoma),
trichodiscoma, medullary thyroid carcinoma, and pulmonary cysts and spontaneous pneumothorax.
(C) Neurofibromatosis Type II is characterized by autosomal dominant inheritance, mutation in NF-2 gene encoding
merlin/schwannomin, bilateral acoustic schwannomas, meningiomas, gliomas, juvenile posterior subcapsular
lenticular opacity.
(D) Buschke-Ollendorf Syndrome is characterized by autosomal dominant inheritance, mutation in LEMD3 gene,
osteopoikilosis, dermatofibrosislenticularisdisseminata, and connective tissue nevi.
(E) Peutz-Jeghers Syndrome is characterized by autosomal dominant inheritance, gene mutation in STK11/LKB1
(serine threonine kinase tumor suppressor gene), perioral, intraoral and acrallentigines, gastrointestinal polyps
with bleeding/intussusception, gastrointestinal adenocarcinomas, and increased frequency of ovarian, breast, and
pancreatic carcinoma.

50. Which of the following is not a factor contributing to the exacerbation of Darier Disease?
A. Occlusive Clothing
B. Anti-Epileptics
C. Sweating
D. Bacterial Infections
E. Viral Infections

Correct answer: (B) Anti-Epileptics.


Explanation: Anti-epileptics have not been know to exacerbate Darier disease, however Lithium, a medication used
in the treatment of Bipolar Disorder, is a medication that can worsen the clinical findings in Darier Disease.
(A) (C) (D) Sweat and occlusive clothing can predispose Darier patients to secondary infections with dermatophytes
and/or bacteria, leading to more malodorous papules and plaques as well as the transformation to vegetating
plaques.
(E) As in atopic dermatitis, viral infections such as the Herpes Simplex Virus, can lead to Kaposi’s Varicelliform
eruption requiring oral or IV anti-viral therapy. Lesions in Kaposi’s Varicelliform Eruption with underlying Darier
Disease, lean toward more hemorrhagic crusting and vesiculation, and be accompanied by fever and malaise.

51. Porphyrias are classified as an hereditary genetic metabolic disorder. Porphyria cutanea tarda is the most
common type of porphyria and is known for photosensitivity, cutaneous findings in sun-exposed areas
including: skin fragility, erosions, crusts, vesicles and bullae, milia, scarring, hyperpigmentation and
hypertrichosis. What is the enzyme in the heme biosynthetic pathway that is deficient in this disease?
A. Aminolevulinic acid dehydratase
B. Porphobilinogen deaminase
C. Uroporphyrinogen III synthase
D. Uroporphyrinogen decarboxylase
E. Protoporphyrinogen oxidase

24
Correct answer: (D) Uroporphyrinogen decarboxylase.
Explanation: Porphyrias are classified as an hereditary genetic metabolic disorder. Porphyria cutanea tarda is the
most common type of porphyria and is known for photosensitivity, cutaneous findings in sun-exposed areas
including: skin fragility, erosions, crusts, vesicles and bullae, milia, scarring, hyperpigmentation and hypertrichosis.
It is caused by a mutation in the enzyme uroporphyrinogen decarboxylase (UROD) in the heme biosynthetic
pathway that is deficient in this disease. When aminolevulinic acid (ALA) dehydratase is deficient, it causes ALA
dehydratase deficiency porphyria. If porphobilinogen deaminase is deficient, this results in acute intermittent
porphyria. When uroporphyrinogen III synthase is deficient, the patient will suffer from congenital erythropoietic
porphyria. And finally, variegate porphyria is caused by a deficiency in protoporphyrinogen oxidase.

52. You are asked to provide assistance with a patient during a clinical conference. Several clinical photos
demonstrating a young patient with limb overgrowth and a large geographic capillary malformation are shown
followed by a video of a Doppler ultrasound showing a fast-flow vascular anomaly. You have ruled out the
presence of a venous malformation, however, the issue of genetic testing has come up. Which genetic mutation
would most likely be present for the patient in question?
A. PTEN
B. AKT1
C. RASA1
D. PIK3CA
E. GLMN

Correct answer: (C) RASA1.


Explanation: The description of the patient in the vignette is most consistent with Parkes-Weber Syndrome or
Capillary Malformation-Arteriovenous Malformation Syndrome (CM-AVM). The presences of a cutaneous capillary
malformation and a fast-flow arteriovenous malformation, as well as the absence of a venous malformation,
make Parkes-Weber and CM-AVM syndrome the most likely diagnosis. RASA1 mutations are responsible for these
syndromes, and testing for them is recommended.
(A) A mutation in the PTEN gene gives rise to the PTEN Hamartoma Syndrome. This syndrome encompasses both
Cowden syndrome and Bannayan-Riley-Ruvalcaba syndrome (BRRS). Both share the common feature of associated
high-flow vascular malformations with increased adipose tissue. Cowden syndrome is characterized by cutaneous
hamartomas, trichilemmomas, sclerotic fibromas and internal malignancies (breast, thyroid). BRRS is associated
with macrocephaly, developmental delay and penile lentigines.
(B) Mutations in the AKT1 gene are responsible for Proteus syndrome, and the more recently described
Macrocephaly-Capillary Malformation Syndrome (M-CM). Proteus syndrome requires a mosaic distribution of its
cutaneous and visceral features for a definitive diagnosis. Most common are the cerebriform connective tissue nevi
on the palms and soles, epidermal nevi, lipomas, slow-flow vascular malformations, and asymmetric
disproportionate overgrowth.
(D) PIK3CA mutations give rise to both CLOVES syndrome (congenital lipomatous overgrowth, vascular
malformations, epidermal nevi, and skeletal anomalies including scoliosis), as well as the Macrocephaly-Capillary
Malformation syndrome (M-CM). The vascular malformations in CLOVES can range from slow flow capillary
malformations to fast flow arteriovenous malformations. Of note, mutations in both PIK3CA and AKT1 can lead to
M-CM. Additionally, aside from Choice (E), many of these overgrowth syndromes share similar features and are
often included in the differential diagnosis of one another. It is therefore important to be able to identify the key
features of each of these syndromes in order to delineate them.
(E) GLMN is the mutation that gives rise to familial glomulovenous malformations.

25
53. A 16 year-old patient with no significant past or family medical history, presents to your clinic with a
malodorous rash of several months, consisting of unilateral whorls composed of red to brown “greasy”
appearing papules affecting the right anterior trunk and right arm only. His presentation is most likely due to:
A. A Heterozygous mutation in ATP2A2
B. A Homozygous mutation in ATP2A2
C. A Post-Zygotic mutation in ATP2A2
D. A Frameshift mutation in ATP2A2
E. A Missense mutation in ATP2A2

Correct answer: (C) A Post-Zygotic mutation in ATP2A2.


Explanation: This vignette clinically describes Type 1 Segmental Darier Disease resulting from a post-zygotic
mutation in the ATP2A2. The characteristic lesions of Darier disease distributed in “whorls” is telling of the typical
Blaschkoid distribution often demonstrated in post-zygotic mutations. Blashckoid lines are invisible lines that
resemble the pattern of cutaneous development and migration during embryogenesis. This is also referred to as a
mosaic pattern and can be seen in other cutaneous disorders such as epidermal nevi, lichen striatus, and
incontinentia pigmenti. The mutation responsible for the changes observed clinically in mosaicism, take place
following fertilization, and affect only one particular cell line and that cell lines progeny but no other cells in a
given embryo. The cell line affected in Type 1 Segmental Darier Disease will harbor the ATP2A2 mutation, but
neighboring cell lines will not. Each cell line is genetically similar, however one carries the mutation. Several other
autosomal dominant single-gene disorders, like Darier Disease, can also be segmental and present with a mosaic
or blaschkoid pattern and include Hailey-Hailey Disese, Neurofibromatosis Type I, and linear porokeratosis. (See
Chapter 62 in Bologna 3rd ED table 62.2 page 945 for a more detailed list).
(A) (B) (D) (E) These would not lead to the clinical presentation detailed in this vignette.

54. Which of the following is a radiologic finding seen in Conradi-Hunermann syndrome?


A. Clinodactyly
B. Chondrodysplasia punctata
C. Bilateral posterior iliac horns
D. Thickening of calvarium
E. Hyperostosis of external auditory canal

Correct answer: (B) Chondrodysplasia punctata.


Explanation: Chondrodysplasia punctata is seen in Conradi-Hunermann syndrome.
(A) Clinodactyly (curvature of the fifth finger toward the adjacent fourth finger) is seen in Cornelia de Lange
syndrome, Russell-Silver syndrome, and Down’s syndrome.
(C) Bilateral posterior iliac horns are seen in Nail-patella syndrome.
(D) Thickening of calvarium is seen in Clouston Syndrome.
(E) Hyperostosis of external auditory canal is seen in Proteus Syndrome.

55. What lab abnormality would you expect in this patient?

A. Decreased urinary uroporphyrin


B. Decreased fecal isocoproporphyrin
C. Increased ferritin
D. Increased RBC porphyrins
E. Increased uroporphyrinogen decarboxylase

26
Correct answer: (C) Increased ferritin.
Explanation: Increased ferritin is seen in porphyria cutanea tarda.
(A) Decreased urinary uroporphyrin is not seen in porphyria cutanea tarda. Instead, it is increased.
(B) Decreased fecal isocoproporphyrin is not seen in porphyria cutanea tarda. Instead, it is increased.
(D) Increased RBC porphyrins are not seen in porphyria cutanea tarda. Instead, it is often within normal range.
(E) Increased uroporphyrinogen decarboxylase is not seen in porphyria cutanea tarda. Catalytic activity of
uroporphyrinogen decarboxylase is decreased in this disorder.

56. Varicose veins, leg ulcers, tall stature, gynecomastia, small testes and infertility all found in the same patient are
characteristics of which of the following choices?
A. Turner syndrome
B. Klinefelter syndrome
C. Trisomy 21
D. Phylloid hypomelanosis
E. Pallister–Killian syndrome

Correct answer: (B) Klinefelter syndrome.


Explanation: Klinefelter syndrome is caused by a chromosomal aberration: 47, XXY (an extra X). Therefore, the
patients are male (XY), but with an additional X (mnemonic: Kline felt her - the X). The syndrome involves, among
other things, varicose veins, leg ulcers, tall stature, gynecomastia, small testes and infertility. Turner syndrome is
caused by another chromosomal aberration: 45, XO (a missing X). Characteristics include: congenital lymphedema
(that commonly resolves by age 2 years), low posterior hairline, numerous melanocytic nevi, small hyperconvex
nails, tendency to form keloids, short stature, webbed neck, “shield-like” chest with widely spaced nipples, absence
of sexual development, cardiovascular defects, renal malformations, sensorineural hearing loss and autoimmune
hypothyroidism. Trisomy 21 (Down syndrome) results from a the following chromosal aberration: 47, XY, +21 (for
boys) or 47, XX, +21 (for girls). Findings include: a single palmar crease, protruding scrotal tongue, cheilitis,
multiple syringomas (often found with calcification), alopecia areata, ichthyosiform hyperkeratosis, keratosis
pilaris, milia-like calcinosis cutis, elastosis perforans serpiginosa, multiple collagenomas, cutis marmorata and
acrocyanosis, a predisposition to developing folliculitis, dermatophyte infections and Norwegian scabies,
vesiculopustular eruptions associated with neonatal transient myeloproliferative disorder, brachycephaly,
characteristic facial appearance (epicanthal folds, at nasal bridge), brush eld spots (speckled irides), broad hands,
brachydactyly, atlantoaxial instability, mental retardation, immunodeficiency, congenital heart defects, duodenal
atresia, leukemia (especially acute myeloid leukemia), hearing loss and autoimmune hypothyroidism. Phylloid
hypomelanosis corresponds to a mosaic trisomy 13. It includes round, oval or oblong hypopigmented macules
resembling oral ornaments as well as absence of corpus callosum. Other features include : retinal coloboma,
craniofacial defects, brachydactyly, clinodactyly, camptodactyly and conductive hearing loss. Pallister–Killian
syndrome is a mosaic tetrasomy 12p (isochromosome 12p) with hypopigmented streaks and swirls following
Blaschko’s lines, fine, sparse hair during infancy, postnatal growth deficiency, characteristic facies (anteverted
nostrils, down-turned mouth with thin upper lip) and mental retardation.

57. A condition with adenosine deaminase deficiency is associated with:


A. Chronic Granulomatous Disease
B. Wiskott-Aldrich Syndrome
C. Severe Combined Imunodeficiency (SCID)
D. Job Syndrome
E. Familial Chronic Mucocutaneous Candidiasis

27
Correct answer: (C) Severe Combined Imunodeficiency (SCID).
Explanation: Severe Combined Imunodeficiency (SCID) is inherited in an autosomal recessive fashion and is
characterized by adenosine deaminase gene mutation leading to accumulation of adenosine, which is toxic to
immature lymphocytes.
(A) Chronic Granulomatous Disease is secondary to nicotinamide dehydrogenase phosphate (NADPH) oxidase
enzyme deficiency.
(B) Wiskott-Aldrich Syndrome is secondary to WAS gene mutations that lead to impaired T-cell activation and
natural killer cell function.
(D) Job Syndrome, also known as Hyper-IgE Syndrome, is inherited in an autosomal recessive fashion and is
secondary to a gene mutation in STAT3 (autosomal dominant) or Tyrosine Kinase 2 (TYK2) (autosomal recessive).
(E) Familial Chronic Mucocutaneous Candidiasis is secondary to intercellular adhesion molecule-1 (ICAM-1)
deficiency. This is not "pure" FCMC but rather the familial chronic nail candidiasis subset of patients, and it has
been associated with low serum levels of ICAM1. It is important to know that "pure" FCMC is associated with the
IL-17/IL-17R/STATE genes. Also there is the APECED syndrome associated with AIRE gene.

58. Which of the following diseases is associated with an IgA plasma cell dyscrasia?
A. Pyoderma gangrenosum
B. Schnitzler’s syndrome
C. Necrobiotic xanthogranuloma
D. Scleromyxedema
E. Scleredema

Correct answer: (A) Pyoderma gangrenosum.


Explanation: There are many dermatological diseases associated with the development of a monoclonal
population of plasma cells. Pyoderma gangrenosum is a neutrophilic disorder associated an IgA monoclonal
gammopathy. Of note, Sneddon-Wilkinson disease (subcorneal pustular dermatosis) is another neutrophilic
disorder associated with an IgA monoclonal gammopathy.
(B) Schnitzler’s syndrome is characterized by an IgM gammopathy, as well as urticaria, arthritis and recurrent fevers.
(C) Necrobiotic xanthogranuloma is a non-langerhan’s cell histocytosis, it is almost always involved with an IgG κ
or λ light chain paraproteinemia.
(D) Scleromyxedema is a disorder characterized by excessive mucin deposition, it is associated with a monoclonal
IgG with a predominance of lambda light chains. (E) Scleredema is a mucinous dermatosis associated with an IgG
gammopathy, IgG κ > IgG λ.

59. What is the syndrome seen in the following picture?

A. Lipoid Proteinosis
B. Pseudoxanthoma Elasticum
C. Buschke-Ollendorf Syndrome
D. Focal Dermal Hypoplasia
E. Familial Partial Lipodystrophy

28
Correct answer: (C) Buschke-Ollendorf Syndrome.
Explanation: Buschke-Ollendorf Syndrome is characterized by autosomal dominant inheritance, mutation in
LEMD3 gene, osteopoikilosis, dermatofibrosis lenticularis disseminata, and connective tissue nevi.
(A) Lipoid Proteinosis is characterized by autosomal recessive inheritance, extracellular matrix protein 1(ECM1)
gene mutation, yellow papular “string of pearls” over eyelids, papules on face, neck and extremities, verrucous
nodules of elbows and hands, bean-shaped hippocampal calcification, large wooden tongue, and hoarse voice.
(B) Pseudoxanthoma Elasticum is characterized by autosomal recessive or dominant inheritance, ABCC6 gene
mutation, yellow papules on sides of neck, axillae, anticubital fossae, abdomen, groin, thighs (also described as
“plucked chicken” skin appearance), angioid streaks and rupture in Bruch’s membrane, gastric artery hemorrhage,
myocardial infarction, stroke, and increased first trimester miscarriage.
(D) Focal Dermal Hypoplasia, also known as Goltz Syndrome, is characterized by X-linked dominant inheritance,
atrophic, hyperpigmented or hypopigmented Blaschkoid streaks, fat herniations along Blaschko’s Lines,
osteopathia striata, oral papillomas, lobster claw deformity of hands, colobomas, alopecia, and syndactyly.
(E) Familial Partial Lipodystrophy is characterized by autosomal dominant inheritance, LMNA gene mutation,
symmetric lipoatrophy of trunk/limbs, acanthosis nigricans, hypertriglyceridemia, and eruptive tuberous
xanthomas.

60. A 12-year-old presents with a lesion on her arm (see image). Physical exam is also significant for digital
abnormalities. There are no thrills noted. This condition may present with all of the following, except?

A. Port-wine stain
B. Limb hemihypertrophy
C. Lymphedema
D. Arteriovenous fistula
E. None. All of the above are correct.

Correct answer: (D) Arteriovenous fistula.


Explanation: Klippel–Trénaunay syndrome (KTS or KT) is a rare congenital condition in which blood vessels and/or
lymph vessels fail to develop properly. The three main features are nevus flammeus (also called port-wine stain),
venous and lymphatic malformations, and soft-tissue hypertrophy of the affected limb. Of note digital anomalies
are present in KTS but not in Parkes-Weber syndrome.
(D) AV fistulas are characteristically associated with Parkes-Weber syndrome, which has many similarities to KTS
otherwise. Mutations on the RASA1 (RAS p21 protein activator 1) gene located on chromosome 5 cause Parkes
Weber Syndrome.
(A) Capillary malformations (Port-wine stains) appear early in Klippel-Trenaunay syndrome (KTS).
(B) Ipsilateral limb hypertrophy and excessive growth of underlying soft tissues is highly characteristic of KTS.
(C) Lymphedema commonly occurs in KTS.

29
61. The patient in the following question has abnormal susceptibility to which of the following infectious organisms?

A. Malassezia furfur
B. Human papillomavirus (HPV)
C. Staph auerus
D. Herpes simplex (HSV)
E. Pseudomonas

Correct answer: (B) Human papillomavirus (HPV).


Explanation: The patient shown has epidermodysplasia verruciformis, characterized by autosomal recessive
inheritance, mutation in EVER1/2 genes, with abnormal susceptibility to HPV infection and increased squamous
cell cancers.

62. Which of the following is the earliest cutaneous manifestation in Tuberous Sclerosis?
A. Connective Tissue Nevus
B. Koenen Tumors
C. Hypomelanotic macules
D. Adenoma Sebaceum
E. Enamel pits

Correct answer: (C) Hypomelanotic macules.


Explanation: Hypomelanotic, or Ash-Leaf, macules are often the first cutaneous manifestation of Tuberous
Sclerosis (TS). These hypopigmented macules (as opposed to the depigmented macules of Vitiligo) are often
present at birth. Although they serve as an early clue to the diagnosis of TS and can be present at birth, they are
often subtle and can be missed in fairer skinned patients. Patients can present with one or several and up to a 100
hypopigmented macules, and 90% of TS patients have more than one. These may also appear as small or guttate
hypopigmented macules over the extremities and are sometimes called confetti macules.
(A) The connective tissue nevus, also called the “Shagreen Patch,” is often not seen until around 2 years of age. It
has a predilection for the trunk and more specifically the lumbosacral area, and can be hypo or hyperpigmented. It
has potential to grow radially and the depressed follicular openings of the involved skin sometimes give it a
“pigskin” texture.
(B) Koenen tumors, which are subungual or periungual angiofibromas, tend to appear later around the age of 4
and favor the lower extremities.
(D) Adenoma sebaceum, or facial angiofibromas, typically first appear around the age of 2, and initially present
with mild erythema before developing into soft pink papules. They are distributed in a bilateral and symmetric
pattern.
(E) Enamel pits, while not a specific finding of TS, can be seen in greater number in TS patients when compared to
the normal population. The presence of many enamel pits may be suggestive of TS but alone, is not enough to
make the diagnosis. These manifest along with the eruption of the primary teeth.

30
63. A 32 year-old-patient presents to your clinic with warty skin colored papules on the dorsum of his feet. They
have been present 5-6 years with no associated symptoms. He states that he has undergone numerous
treatments with liquid nitrogen, topical imiquimod, and even candida antigen immunotherapy injections with
no resolution. The remainder of the cutaneous exam, including face, chest, oral mucosa, and nails was
unremarkable. Which of the following mutations is responsible for this patient's presentation?
A. ATP2C1
B. ATP7A
C. ATP7B
D. ATP2A2
E. ATP2A1

Correct answer: (D) ATP2A2.


Explanation: The patient in the vignette presents with signs consistent with Acrokeratosis Verruciformis of Hopf.
The mutation responsible for this condition presenting with skin colored to pink smooth to occasionally verrucous
colored papules typically on the dorstal aspects of the distal extremities is the same mutation responsible for
Darier-White, or Darier Disease. Patient’s with Darier Disease may have similar findings, however, involvement of
the sebaceous areas of the skin, the hairline, the posterior scalp, and sometimes intertriginous areas with single
red-brown papules which can coalesce in to larger warty plaques plaques (+/- odor depending on bacterial
colonization), is a prominent finding. Additionally, patients with Darier disease may have findings such as
longitudinal erythronychia (red/white striped nails) distal V-nicking, and/or mucosal involvement. Patients with
Acrokeratosis verruciformis of Hopf do not present with these findings.
(A) ATP2C1 – Hailey-Hailey Disease (common mnemonic “I want 2C1 Hailey’s Comet).
(B) ATP7A – Protein responsible for Copper transport and mutated in Menke’s Kinky Hair Syndrome, as well as the
Occipital Horn Syndrome.
(C) ATP7B – Protein is another Copper Dependent ATPase mutated in Wilson’s Disease (Hepatolenticular
Degeneration).
(E) Distractor.

64. Which of the following capillary malformations would be most responsive to therapy with a Pulsed-Dye Laser?
A. A Port-Wine Stain in a V2 distribution in Type II Skin
B. A Port-Wine Stain in a V1 distribution in Type V Skin
C. A Port-Wine Stain on the leg in Type II Skin
D. A Port-Wine Stain in a V1 Distribution in Type I Skin
E. A Port-Wine Stain in a V2 Distribution in Type IV Skin

Correct answer: (D) A Port-Wine Stain in a V1 Distribution in Type I Skin.


Explanation: Pulsed-Dye Laser (PDL) therapy has been the gold standard in the treatment of capillary
malformations, including port wine stains (PWS) for the past few decades. Certain factors have been studied that
can better able the clinician to predict outcomes during treatment. Factors that increase the efficacy and improve
outcomes in PWS being treated by PDL include; Fitzpatrick Type I or II skin, PWS of the face, and/or neck. Factors
that decrease the efficacy of PDL in the treatment of capillary malformations, including PWS, are those that are
located in a V2 distribution, port wine stains located on the extremities, and port wine stains in darker skinned
patients (Fitzpatrick Types IV-VI).

31
65. You are a reviewing a patient’s chart who you suspect has Klippel-Trenaunay Syndrome (KTS). She has had a
significant work-up done including visits to an orthopedic surgeon, radiologist, and now dermatologist. You
have pulled up her imaging in order to determine if your patient’s presentation is consistent with KTS. Which
of the following is NOT a feature of Klippel-Trenaunay Syndrome?
A. Geographic capillary malformation
B. Lymphatic malformation
C. Venous malformation
D. Limb length discrepancy
E. Arteriovenous malformation

Correct answer: (E) Arteriovenous malformation.


Explanation: Arteriovenous malformations are not part of the spectrum of Klippel-Trenaunay Syndrome. The
remaining choices (A-D) are the primary features of KTS. The combination of vascular stain, limb overgrowth, and
a fast-flowing vascular anomaly, such as a arteriovenous malformation raises concern for a separate entity known
as Parkes-Weber Syndrome which carries a different prognosis from KTS due to the frequent complication of high-
output cardiac failure. The presence or absence of a lymphatic malformation is sometimes a factor used to
delineate complex from simple KTS respectively. While other classification systems require the presence of
lymphatic malformations, geographic capillary malformations as well as significant venous anomalies to make
that distinction. Not alllower extremity capillary malformations are associated with venous malformations so care
should be taken to appropriately assess patients presenting with lower limb port-wine stains before rendering a
diagnosis of KTS.

66. A patient presents with photodistributed erythema and hyperpigmentation, cerebellar ataxia, mild intellectual
disability, aminoaciduria. Which of the following is a treatment for this syndrome?
A. Vitamin C
B. Ferrochelatase
C. Zinc
D. Nicotinic Acid
E. Vitamin D

Correct answer: (D) Nicotinic Acid.


Explanation: Hartnup disease is characterized by autosomal recessive inheritance, faulty transport of amino acids
in small intestine and kidney causing decreased absorption of tryptophan, photodistributed erythema and
hyperpigmentation, cerebellar ataxia, mild intellectual disability, and psychiatric disturbances. Patients should
have nicotinic acid supplementation and eat a high protein diet.

67. In Junctional Epidermolysis Bullosa where does the skin split?


A. Epidermal granular layer
B. Epidermal basal layer
C. Basement Membrane at Lamina Lucida
D. Dermal Lamina Densa
E. Dermal Sublamina Densa

Correct answer: (C) Basement Membrane at Lamina Lucida.


Explanation: Basement membrane split at the level of the Lamina Lucida is seen in Junctional Epidermolysis
Bullosa.
(B) Epidermal basal layer split is seen in Epidermolysis Bullosa Simplex.
(E) Dermal Sublamina Densa split is seen in Dystrophic Epidermolysis Bullosa.

32
68. What is the second most common neoplasm in Gorlin’s syndrome?
A. Basal Cell Carcinoma
B. Squamous Cell Carcinoma
C. Melanoma
D. Medulloblastoma
E. Ovarian Fibrosarcoma

Correct answer: (D) Medulloblastoma.


Explanation: Gorlin’s syndrome, also known as Basal Cell Nevus Syndrome, is characterized by autosomal
dominant inheritance, mutation in PTCH1 gene coding sonic hedgehog membrane receptor protein. Key features
include multiple basal cell carcinomas, palmar pits, jaw cysts, frontal bossing, bifid ribs, calcification of falx cerebri,
agenesis of corpus callosum, hypertelorism, and ovarian fibromas. The most common malignancy is basal cell
carcinoma, followed by medulloblastoma.

69. What is your diagnosis?

A. neonatal varicella
B. neonatal HSV
C. incontinentia pigmenti
D. focal dermal hypoplasia
E. none of the above

Correct answer: (C) Incontinentia pigmenti.


Explanation: Linear erythema and vesicles in a Blaschkoid distribution should raise concern for Incontinentia
Pigmenti (IP). IP is a inherited in an X-linked dominant fashion (usually lethal in males), and is caused by NEMO
gene mutations. IP progresses through stages: inflammatory/vesicular, verrucous, hyperpigmented, and
hypopigmented/atrophic.
(A) Neonatal varicella can present with vesiculopustules, however the distribution would not be characteristically
Blashkoid.
(B) Vesicular lesions of neonatal HSV can mimic vesicular stage incontinentia pigmenti (IP), however the
distibution would not be Blashkoid.
(D) Focal dermal hypoplasia (Goltz syndrome) is another X- linked dominant disorder that presents with skin
lesions distributed in a Blashkoid fashion. However, Goltz syndrome presents with linear atrophy along Blashko’s
lines, not linear vesicular lesions.

70. A patient presents to your clinic with clinical findings consistent with 5+ café au lait macules, axillary freckling,
mild intellectual impairment, and several lipomas. He has no contributory family history, no ocular findings,
and no cutaneous findings consistent with neurofibromas or plexiform neurofibromas. Genetic testing for
Neurofibromatosis Type 1 returned negative. Which of the following gene mutations is responsible for this
patient's condition?
A. Merlin
B. GNAS
C. SPRED-1
D. PTPN11
E. RET

33
Correct answer: (C) SPRED-1.
Explanation: The patient has a diagnosis of Legius Syndrome with the responsible mutation being SPRED-1. This is
another neurocutaneous disorder with prominent CALM’s and intertriginous freckling. Additionally, mild
intellectual disability as well as macrocephaly, are also common features and similar in presentation to NF-1.
Legius syndrome accounts for up to 2% of all patients meeting criteria for NF-1. SPRED-1, similar to neurofibromin,
also inhibits the Ras-Raf pathway and therefore negatively affects the mitogen activated protein kinase (MAPK)
pathway. In contrast to NF-1, patients with Legius Syndrome do not have associated ocular findings such as iris
hamartomas or optic gliomas.
(A) Merlin is mutated in Neurofibromatosis Type 2 which shares CALMs and an absence of Lisch Nodules and
intellectual impairment. Its primary feature is bilateral acoustic schwannomas along with schwannomas as the
most prominent cutaneous feature.
(B) GNAS is mutated in McCune Albright which is most recognizable for its large, geographic, “Coast of Maine”
unilateral CALM. Additional findings include endrocrinopathies including precocious puberty and polyostotic
fibrous dysplasia.
(D) PTPN11 is the mutation responsible for Noonan Syndrome typified by a syndromic facies, short stature and
accompanying cardiac and pulmonary abnormalities. Additionally, LEOPARD syndrome shares the PTPN11
mutation and presents with the following:(Lentigines, EKG conduction abnormalities, Ocular Hypertelorism,
Pulmonic Stenosis/HTN, Abnormal Genitalia, Retarded Growth and Deafness: Sensorineural).
(E) RET is the mutation in several genodermatoses including Multiple Endocrine Neoplasia Type IIa and IIb (MEN).
The latter has cutaneous findings consistent with mucosal neuromas, while the former often has lichen or macular
amyloid as its presenting feature. Both MEN IIa and IIb share medullary thyroid carcinoma and
pheochromocytoma as associated and defining features.

71. Hair shaft abnormalities can be helpful in identifying certain genodermatoses. Tiger-tail hair is a feature of
which of the following?
A. Netherton syndrome
B. Trichothiodystrophy
C. Arthrogryposis–renal dysfunction–cholestasis (ARC) syndrome
D. Mitochondrial disorders
E. AEC, ankyloblepharon–ectodermal dysplasia–clefting

Correct answer: (B) Trichothiodystrophy.


Explanation: Hair shaft abnormalities can be helpful in identifying certain genodermatoses. Trichothiodystrophy
with ichthyosis can also be called: Tay syndrome, sulfur-deficient brittle hair syndrome, IBIDS: ichthyosis, brittle
hair, infertility, developmental delay, short stature and also PIBIDS: photosensitivity plus IBIDS. The disorder is
inherited in an autosomal recessive fashion. Clinical features include short, unruly, fragile hair of the scalp,
eyebrows and eyelashes due to abnormally low sulfur content. Body, axillary and pubic hair may also be affected.
Nail changes are common. Patients with trichothiodystrophy may present at birth or in the neonatal period with
erythroderma and generalized fine scaling. Photosensitivity occurs in half of patients, but there is no propensity to
develop cutaneous malignancies. By light microscopy, hair shafts show transverse fractures (trichoschisis) in
conjunction with an irregular surface and diameter. Polarizing light microscopy is the gold standard for detecting
the characteristic light and dark bands called tiger-tail hair. If photosensitivity is present, sun protection is
essential. Treatment of the ichthyosis includes humectants and in severe cases, systemic retinoids. Trichorrhexis
invaginata is a feature of Netherton syndrome. Pili torti can be found in arthrogryposis–renal dysfunction–
cholestasis (ARC) syndrome, but also in pachyonychia congenita, Menkes disease, Ichthyosis– hypotrichosis
syndrome as well as in mitochondrial disorders and others. In AEC, ankyloblepharon–ectodermal dysplasia–
clefting, lightly pigmented, wiry and coarse hair can be found with patchy alopecia.

34
72. A patient presents with blue lunula, hepatomegaly, and yellow-brown ring-like discoloration of her eye. What is
the defect in this condition?
A. ATPase copper-transporting beta polypeptide
B. phenylalanine hydroxylase
C. acid beta-glucosidase
D. alpha-galactosidase A
E. cystathione beta-synthase

Correct answer: (A) ATPase copper-transporting beta polypeptide.


Explanation: ATPase copper-transporting beta polypeptide is deficient in Wilson disease.
(B) Phenylalanine hydroxylase is deficient in Phenylketonuria.
(C) Acid beta-glucosidase is deficient in Gaucher disease.
(D) Alpha-galactosidase A is deficient in Fabry disease.
(E) Cystathione beta-synthase is deficient in Homocystinuria.

73. This condition is characterized by?

A. Autosomal recessive mode of inheritance


B. Association with Noonan syndrome
C. Association with Rombo syndrome
D. Absence of atrophy and scarring
E. None of the above

Correct answer: (B) Association with Noonan syndrome.


Explanation: Ulerythema ophryogenes has been reported in association with Noonan syndrome, cardio-facio-
cutaneous syndrome, Cornelia de Lange syndrome, and Rubinstein-Taybi syndrome.
(A) Ulerythema ophryogenes (keratosis pilaris atophicans faciei) is a disorder of follicular keratinization that
presents with erythematous follicular papules that lead to eventual scarring alopecia of the lateral third of the
eyebrows. This condition is usually sporadic, although autosomal dominant inheretance has been reported.
(C) Rombo syndrome is associated with atrophoderma vermiculatum, which typically presents as pitted atrophic
depressions on the cheeks.
(D) Ulerythema ophryogenes characteristically leads to atrophic scarring.

74. What is the first finding in patients with tuberous sclerosis?


A. Shagreen patch
B. Hypomelanotic macules
C. Retinal hamartomas
D. Angiomyolipoma
E. Rhabdomyoma

35
Correct answer: (E) Rhabdomyoma.
Explanation: Tuberous sclerosis is inherited in an autosomal dominant fashion with mutations in either TSC1
(encodes hamartin) or TSC2 (encodes tuberin). The earliest finding overall which may be detected on ultrasound
are cardiac abnormalities (present 50-60 % of time) and is most often a cardiac rhabdomyoma. The earliest
dermatological finding most characteristic of tuberous sclerosis are ash-leaf hypopigmented macules. Key
dermatologic features include facial angiofibromas (adenoma sebaceum), collagenomas (Shagreen
patch/connective tissue nevus), periungual fibromas (Koenen’s tumors), and cafe au lait macules. Other features
include gingival fibromas, dental enamel pits, seizures, mental deficits, brain tumors (astrocytomas, cortical
tumors), kidneys (angiomyolipoma), heart (rhabdomyoma), and lungs.

75. This premature baby was noted with the skin findings below at birth. This presentation may appear in
association with which of the following congenital ichthyoses?

A. Lamellar ichthyosis
B. Congenital ichthyosiform erythroderma
C. Sjogren-Larssen syndrome
D. A&B
E. All of the above

Correct answer: (E) All of the above.


Explanation: Choices (A), (B) and (C) are all true statements.
(A) Newborns with lamellar ichythosis commonly present with a collodion membrane.
(B) Newborns with Congenital ichthyosiform erythroderma commonly present with a collodion membrane.
(C) Although rare, collodion membranes can present in association with Sjogren-Larssen syndrome.

76. Eye findings seen in focal dermal hypoplasia include?


A. Comma-shaped corneal opacities
B. Eyelid string of pearls
C. Brushfield spots
D. Retinal hemangioblastomas
E. Coloboma

Correct answer: (E) Coloboma.


Explanation: Colobomas are seen in focal dermal hypoplasia.
(A) Comma-shaped corneal opacities are seen in X-linked ichthyosis.
(B) Eyelid string of pearls are seen in lipoid proteinosis.
(C) Brushfield spots are seen in Down syndrome.
(D) Retinal hemangioblastomas are seen in Von-Hippel-Lindau syndrome.

36
77. Hamartin and Tuberin, the protein products of the TSC1 and TSC2 genes respectively, are important
regulators of the mTOR pathway. Mutations in these proteins are responsible for the phenotype observed in
Tuberous Sclerosis. Which of the following proteins also functions to regulate the activity of mTOR?
A. CDKN2A
B. RET
C. Neurofibromin
D. AKT
E. p53

Correct answer: (D) AKT.


Explanation: Hamartin interacts with Tuberin leading to the dephosphorylation of a GTPase protein called Rheb.
Dephosporylation of Rheb inhibits its functions as a promoter of cell growth and proliferation that would normally
take place following the signaling of mTOR. AKT, the gene mutated in Proteus syndrome, also plays a significant
role in the mTOR signaling pathway. The activation of Akt (also known as Protein Kinase B) is one of the most
frequently observed alterations in human cancer and more specifically tumor cells. Activation of Akt inhibits the
Rheb suppression of both Hamartin and Tuberin and leads to the promotion of cell growth, proliferation, and
ultimately tumor formation.
(A) CDKN2A is the gene mutated in Familial Melanoma.
(B) RET is the gene mutated in both Multiple Endocrine Neoplasia Type IIa and Type IIB.
(C) NF1 is the gene that codes for Neurofibromin, the protein mutated in Neurofibromatosis Type 1.
(E) p53 is a tumor suppressor protein and is mutated in numerous cancers including Squamous Cell Carcinoma.

78. What should be examined at regular intervals for a patient with the following condition?

A. Ophthalmology examination
B. Ultrasound
C. Complete Blood Count
D. Liver Function Testing
E. Limb length and circumference

Correct answer: (E) Limb length and circumference.


Explanation: Limb length and circumference should be measured at regular intervals in CMTC because patients
may have ipsilateral hemiatrophy or hemihypertrophy of extremities. Key findings in CMTC include reticulated
vascular patches on extremities, trunk, face, cardiovascular anomalies such as patent ductus arteriosus, glaucoma,
and intellectual disability. (A) Ophthalmology examination would be appropriate in Cutis Marmorata
Telangiectatica Congenita (CMTC) only if there was periocular involvement.

79. Which of the following syndromes is at an increased risk of developing Wilms Tumor?
A. Beckwith-Wiedemann Syndrome
B. Bloom Syndrome
C. Macrocephaly – Capillary Malformation (M-CM) Syndrome
D. A, B and C
E. A and C only

37
Correct answer: (D) A, B and C.
Explanation: Wilms Tumor is the commonest childhood tumor and has an incidence of 1 in 10,000. Its median age
of diagnosis is 3 and 4 years and 80% of individuals are diagnosed by age 5. This renal tumor is associated with
several syndromes including Beckwith-Wiedemann, Bloom, and M-CM syndrome. Both Beckwith-Widemann and
M-CM syndrome are considered overgrowth syndromes, and share the common feature of capillary malformations
typically located on the glabella. While M-CM’s features are typically macrocephaly in addition to the capillary
malformation, other features include brain and body asymmetry, digital anomalies like syndactyly, joint laxity and
seizures. Mutations in M-CM include AKT3, PIK3CA or PIK3R2 genes. Interestingly, AKT is also mutated in Proteus
syndrome, which is characterized, by overgrowth as well as vascular malformations and connective tissue nevi.
Bloom Syndrome, caused by a mutation in RecQL3. This gene is responsible for a DNA helicase that is important in
the protection against sister chromatid exchange. It is an autosomal recessive disorder with its pathogenesis due
to chromosomal instability. It is characterized by telangiectasia development in sun exposed skin as well as short
stature, characteristic faces, poikiloderma, and immunodeficiency. Although relatively rare, with fewer than 200
cases reported in the literature, Wilms tumor has been found in 6 of those cases, approximately 3%. Serial
abdominal ultrasounds should be recommended for patients with the above listed conditions.

80. The pathogenesis of Hailey-Hailey Disease is due to a mutation leading to abnormal calcium sequestration in
which of the following?
A. Ribosomes
B. Endoplasmic Reticulum
C. Mitochondria
D. Cytoplasmic Lysosomes
E. Golgi Apparatus

Correct answer: (E) Golgi Apparatus.


Explanation: In Hailey-Hailey Disease a mutation in ATP2C1 leads to the abnormal sequestration of calcium in the
Golgi apparatus. This prevents the complete processing of necessary junction proteins required for cell-to-cell
adhesion and leads to the abnormal formation of adherens junctions. This is believed to be the underlying
pathogenesis of the significant acantholysis observed histopathologically in Hailey-Hailey Disease.
(A) (C) (D) These are not involved in Calcium homeostasis leading to the normal maintenance of the epidermis.
(B) The Endoplasmic Reticulum is the sight of abnormal calcium sequestration in Darier disease and is due to a
mutation in ATP2A2. This mutation is more specifically responsible for decreased folding and trafficking of
important proteins, such as desmoplakin, that are required to prevent acantholysis. Mutations in either ATP2A2 or
ATP2C1 can lead to acantholysis, but differ in the respective organelles involved.

81. A 30 year-old patient presents to your clinic with a complaint regarding the presence of the facial papules
pictured below. He has a history of white spots on his back since birth as well as a mild form of epilepsy. He has
found similar pink lesions around his right 3rd toenail but it does not bother him as much as the facial lesions.
All of the following are appropriate treatment options EXCEPT?

A. Dermabrasion
B. Topical Rapamycin
C. Fractional Resurfacing
D. Pulsed Dye Laser
E. All of the above treatment modalities are
appropriate options

38
Correct answer: (E) All of the above treatment modalities are appropriate options.
Explanation: All of the treatment options mentioned have been utilized with varying rates of success in the
treatment of the facial angiofibromas that accompany Tuberous Sclerosis. In addition to the potential for side
effect to some of the destructive methods used in the treatment of angiofibromas, it is important to note that
lesions often recur. More recently, and following the successful treatment of renal angiomyolipomas as well as
subependymal astrocytomas with systemically administered Rapamycin, topical Rapamycin, in a 1% solution or
ointment, has demonstrated efficacy in the treatment of facial angiofibromas in several studies.

82. The skin condition below on an extensor surface may be found all of the following disorders, except?

A. Atopic dermatitis
B. Noonan syndrome
C. Down syndrome
D. Graham-Little-Picardi-Lasseur syndrome
E. None. All of the above are associated with this
skin condition

Correct answer: (E) None. All of the above are associated with this skin condition.
Explanation: All of the above conditions are associated with KP.
(A) Keratosis pilaris appear as monomorphic follicular papules on extensor surfaces, and are highly associated with
atopic dermatitis.
(B) Noonan syndrome is associated with KP.
(C) Down syndrome is associated with KP.
(D) Graham-Little-Picardi-Lasseur syndrome presents with lichen planopilaris the scalp, scarring alopecia of the
scalp, and KP-like skin lesions.

83. A 16 year-old patient with a past medical history of attention deficit disorder presents with numerous tan to
brown patches of varying sizes, all greater then 1.6 cm, as well as several skin colored papules on the back. He
states his father also had similar lesions. What is the function of the protein mutated in this patient’s condition?
A. p53 inhibitor
B. inhibitor of Smoothened
C. enhancer of the Rb protein
D. negative regulator of the Ras protein
E. inhibitor of the mammalian target of rapamycin

Correct answer: (D) Negative regulator of the Ras protein.


Explanation: Neurofibromin, the protein mutated in Neurofibromatosis Type 1, is a negative regulator of the
RAS/RAF/MEK/ERK pathways, and directly inhibits the RAS protein. It functions by dephosphorylating GTP bound
to Ras to GDP, thereby inhibiting its promotion of cell growth and proliferation. The RAS/RAF/MEK/ERK pathways
is important to know as mutations in its pathway are responsible for several genetic diseases such as Noonan and

39
LEOPARD syndrome, as well as NF-1. Additionally, similar mutations have been found in skin cancers such as
cutaneous melanoma.
(A) A – p53 is a mutation commonly found in SCC and BCC, but more commonly in SCC.
(B) Smoothened is mutated in Basal Cell Nevus Syndrome.
(C) Rb, in addition to being mutated in retinoblastoma its namesake, is also found to be mutated in osteosarcoma,
breast carcinoma, melanoma, and in high risk HPV associated carcinoma.
(E) The mammalian target of rapamycin (mTOR) is a protein that is part of the pathway acted on by hamartin and
tuberin (proteins mutated in Tuberous Sclerosus) which contributes to cell growth and proliferation.

84. This child was born with white hair and milky white skin. All of the following mutations are implicated in this
condition, except?

A. Tyrosinase
B. Tyrosinase-related protein-1
C. Membrane-associated transporter protein
D. Pink eyed gene
E. None of the above. All are implicated in the
condition depicted

Correct answer: (E) None of the above. All are implicated in the condition depicted.
Explanation: All of the statements are true.
(A) Oculocutaneous albinism (OCA) type IA and IB are associated with tyrosinase mutations (TYR) which produce
completely inactive (IA) or partially active (IB) tyrosinase enzyme activity. These patients synthesize the least
amount of melanin, and therefore have the most severe phenotype.
(B) TYRP1 gene mutations are implicated in OCA3.
(C) MATP gene mutations are implicated in OCA4, which presents with mild to moderate pigmentary dilution of
skin, hair, and iris.
(D) P gene mutations are implicated in OCA2. Like OCA4, these patients have mild to moderate pigmentary dilution
of skin, hair, and iris.

85. Which disease is associated with increased cholesterol sulfate levels?


A. Lamellar Ichthyosis
B. Non-bullous Congenital Ichthyosiform Erythroderma
C. X-linked Ichthyosis
D. Epidermolytic Hyperkeratosis
E. Ichthyosis Vulgaris

Correct answer: (C) X-linked Ichthyosis.


Explanation: X-linked Ichthyosis is associated with increased cholesterol sulfate levels and is confirmed by serum
lipoprotein electrophoresis.

40
(A) Lamellar Ichthyosis is associated with transglutaminase deficiency.
(B) Non-bullous Congenital Ichthyosiform Erythroderma is associated with transglutaminase deficiency. (
D) Epidermolytic Hyperkeratosis is associated with keratin 1 and keratin 10 gene mutations.
(E) Ichthyosis Vulgaris is associated with decreased/absent profilaggrin.

86. Identify the gene product that is altered in KID (keratitis–ichthyosis–deafness) syndrome in the following choices:
A. Connexin 26
B. Keratin 1
C. 3β-hydroxysteroid dehydrogenase
D. Cathepsin C
E. Phytanoyl-CoA 2-hydroxylase

Correct answer: (A) Connexin 26.


Explanation: KID (keratitis–ichthyosis–deafness) syndrome has an autosomal dominant mode of inheritance and is
caused by a mutation in GJB2 (gap junction protein β2) (gene) and affects Connexin 26 found in the eye, inner ear
and skin. Connexin26 is also involved in Palmoplantar keratoderma with deafness as well as in Bart-Pumphey
syndrome.Epidermolytic ichthyosis (bullous congenital ichthyosiform erythroderma) has an autosomal dominant
mode of inheritance and is caused by a mutation in KRT1 or 10 (genes) resulting in a Keratin 1 or 10 (gene
products) that are altered.
CHILD syndrome has an X-linked dominant mode of inheritance and is caused by a mutation in NSDHL (gene) and
results in problems expressing 3β-hydroxysteroid dehydrogenase (gene product).Papillon–Lefèvre syndrome and
Haim–Munk syndrome both have an autosomal recessive.
Cathepsin C mode of inheritance and are both caused by a mutation in CTSC (gene) which encodes Cathepsin C.
Refsum disease has an autosomal recessive mode of transmission and is caused by a mutation in one of the
following : PHYH or PAHX both encoding Phytanoyl-CoA 2-hydroxylase or a mutation in PEX7 encoding Peroxin-7.

87. A patient presents to your office with cutaneous signs and symptoms. You find localized lesions in sun-exposed
areas including: skin fragility, blistering, erosions, scarring. No neurological signs or symptoms are reported by
the patient. He does not report the use of any medication. What should you measure?
A. Measure stool porphyrins
B. Measure urinary porphyrin precursors aminolevulinic acid and porphobilinogen
C. Measure zinc-chelated protoporphyrin
D. Measure erythrocyte protoporphyrin
E. Measure urinary porphyrins uroporphyrin and coproporphyrin

Correct answer: (E) Measure urinary porphyrins uroporphyrin and coproporphyrin.


Explanation: When a patient presents cutaneous signs and symptoms in sun-exposed areas such as skin fragility,
blistering, erosions, scarring, a screening should be performed including urinary porphyrins uroporphyrins and
coproporphyrins. If the results are normal, in a symptomatic patient, the diagnosis of porphyria is excluded.
Consideration of pseudoporphyria and epidermolysis bullosa acquisita should then be considered.
Incorrect answers:
(A) If the results are positive cutaneous porphyria is confirmed. Then, a measurement of stool porphyrins can be
performed to differentiate between porphyria cutanea tarda (isocoproporphyrin), variegata porphyria and
hereditary coproporphyria.
(B) If acute attacks occur, measuring urinary porphyrin precursors urinary porphyrin precursors aminolevulinic acid
(ALA) and porphobilinogen (PBG) are useful. If ALA or PBG are elevated, measuring stool porphyrins will allow the
differentiation between acute intermittent porphyria , variegate porphyria, and hereditary coproporphyria.

41
(C) To differentiate between erythropoietic protoporphyria and X-linked dominant protoporphyria, a measurement
of zinc-chelated protoporphyrin can be performed, which shows more predominance in the former.
(D) Measuring erythrocyte protoporphyrin and obtaining elevated results orient towards erythropoietic
protoporphyria and X-linked dominant protoporphyria.

88. What next test should be performed on the following patient?

A. Blood pressure
B. Urinalysis
C. EKG
D. Spirometry
E. Pulse Oximetry

Correct answer: (A) Blood pressure.


Explanation: Neurofibromatosis I is characterized by autosomal dominant inheritance of a gene mutation in NF-1,
involved in tumor suppression. Key features include café au lait macules, neurofibromas, axillary/inguinal freckling
(Crowe’s sign), optic glioma, Lisch nodule (iris hamartoma), sphenoid dysplasia, learning disabilities, increased risk
of nonlymphocytic childhood leukemia and seizures. Patients should have their blood pressure checked to evaluate
for hypertension. In children, renal artery stenosis may be the cause. In adults, pheochromocytoma may be
present. The NIH consensus criteria for diagnosis of Neurofibromatosis I require two or more features of the
following: 6+ café au lait macules >5mm in greatest diameter in prepubertal individuals and over 15mm in
postpubertal individuals, 2+ neurofibromas or one plexiform neurofibroma, axillary or inguinal freckling, optic
glioma, 2+ Lisch nodules, osseous lesion such as sphenoid dysplasia or thinning of long bone cortex with or without
pseudoarthrosis, a first-degree relative with NF-1.

89. Nevus spilus is found in which forms of phakomatosis pigmentovascularis?


A. Type I
B. Type II
C. Type III
D. Type IV
E. Types III and IV

Correct answer: (E) Types III and IV.


Explanation: Phakomatosis pigmentovascularis type I consists of capillary malformation + epidermal nevus. Type II
consists of capillary malformation + dermal melanocytosis (+/- nevus anemicus). Type III consists of capillary
malformation + nevus spilus (+/- nevus anemicus). Type IV consists of capillary malformation + dermal
melanocytosis + nevus spilus (+/- nevus anemicus).

42
90. What eye finding is most common in a patient with Marfan Syndrome?
A. Cataracts
B. Keratoconus
C. Glaucoma
D. Ectopia Lentis

Correct answer: (D) Ectopia Lentis.


Explanation: Marfan Syndrome is characterized by autosomal dominant inheritance, mutation in fibrillin gene, tall
stature, arachnodactyly, pectus excavatum, high-arched palate, loose joints, kyphoscoliosis, dilation of ascending
aorta and regurgitation, congestive heart failure, mitral valve prolapse, aortic dissection and rupture, and
elastosis perforans serpiginosa. Both myopia and ectopia lentis are seen in Marfan’s Syndrome, but ectopia lentis
is more common.

91. The condition seen in the picture may be seen in which of the following genodermatoses?

A. Alkaptonuria
B. Phenylketonuria
C. Gaucher Disease
D. Niemann-Pick DIsease
E. Fabry Disease

Correct answer: (E) Fabry Disease.


Explanation: Fabry Disease, also known as angiokeratoma corporis diffusum, is an X-linked recessive alpha-
galactosidase A (GLA) gene mutation resulting in accumulation of glycosphingolipids particularly in heart, kidney,
and vascular endothelium eventually leading to myocardial infarction and renal failure. Angiokeratomas are
symmetrically distributed between the umbilicus and knees, oral mucosa, and conjunctiva. Other key features
include whorled corneal opacities, hypohidrosis, “maltese crosses” in urine, and pain crises/acroparesthesias most
severe on hands and feet.
(A) Alkaptonuria is an autosomal recessive homogentisate oxidase (HGO gene) deficiency causing ochronosis,
brown/black cerumen and sweat, large joint arthropathy, dark urine, and valvular heart disease.
(B) Phenylketonuria is an autosomal recessive phenylalanine hydroxylase (PATH gene) deficiency. Phenylalanine
and phenylalanine metabolite accumulation inhibits tyrosine in melanogenesis and has toxic effects on the CNS.
Key features include generalized hypopigmentation (blond hair, blue eyes), eczema, mousy odor, seizures, and
intellectual disability.
(C) Gaucher Disease is an autosomal recessive inherited beta-glucosidase (GBA) gene mutation. Decreased
glucocerebrosidase causes build-up of glucocerebroside in histiocytes (Gaucher’s cells) with “crumpled tissue
paper” appearance. Key features include hepatosplenomegaly, diffuse hyperpigmentation on face, neck, hands,
petechiae, ecchymoses, CNS involvement, aseptic necrosis of femoral head, and erlenmeyer flask bone deformity.
(D) Niemann-Pick Disease is an autosomal recessive sphingomyelin phosphodiesterase -1 (SMPD-1) gene mutation
resulting in “foamy” accumulation of sphingomyelin in histiocytes. Key features include xanthomas,
hepatosplenomegaly, thrombocytopenia, failure to thrive, progressive CNS deterioration/dystonia, dysarthria, and
cherry red spot in fovea.

43
92. All of the following are skin findings in Ataxia-Telangiectasia (AT) EXCEPT?
A. Hair greying and canities
B. Café Au Lait Macules
C. Eczematous Dermatitis
D. Discrete pink scaly plaque with thick adherent scale
E. Seborrheic Dermatitis

Correct answer: (D)


Explanation: Discrete pink scaly plaque with thick adherent scale are not a feature of ataxia-telangiectasia. They
are associated with psoriasis.
(A) (B) Hair greying (A), canities, and progeroid changes of the skin are also common in patients with AT.
Subcutaneous fat loss, accompanied by atrophic and sclerotic facial skin is noted in almost 90% of patients. Hyper-
and hypopigmentation, as well as café au lait macules (B) are also often seen in children with AT.
(C) (E) Finally, as witnessed in other immunodeficiencies, eczematous and seborrheic dermatitis are a common
finding in patients with AT. Other cutaneous manifestation of AT include poikiloderma, keratosis pilaris, warts,
vitiligo, hirsutism, and acanthosis nigricans.

93. Which of the following is false about the syndrome seen in the following picture?

A. Doppler ultrasound is an appropriate diagnostic


test in childhood
B. Inheritance is autosomal recessive
C. Superficial thrombophlebitis is a complication
D. Compression stockings are a useful intervention
E. Deep vein thrombosis and pulmonary embolism
are rare

Correct answer: (B) Inheritance is autosomal recessive.


Explanation: Klippel Trenaunay Syndrome is a sporadic vascular malformation of a limb with bone and soft tissue
hypertrophy. Key features include deep venous malformations, superficial venous varicosities, superficial
thrombophlebitis, deep vein thrombosis and pulmonary embolism rarely, and lymphatic malformation.

94. A patient presents with recurrent otitis media and atopic dermatitis. What syndrome may the child have?
A. Chronic Granulomatous Disease
B. Wiskott-Aldrich Syndrome
C. Severe Combined Immunodeficiency (SCID)
D. Familial Chronic Mucocutaneous Candidiasis

44
Correct answer: (B) Wiskott-Aldrich Syndrome.
Explanation: Wiskott-Aldrich Syndrome is characterized by X-linked recessive inheritance of a mutation in WASP, a
protein important in signal transduction and actin filament assembly, resulting in impaired T-cell activation and
NK cell function. Key features include atopic dermatitis, recurrent bacterial infections (particularly encapsulated
organisms) with otitis media, meningitis, pneumonia, thrombocytopenia with petechiae/purpura/epistaxis,
increased lymphoreticular malignancy (especially non-Hodgkin’s lymphoma), and increased IgA/D/E and
decreased IgM.

95. A newborn 3 week-old male is brought to your clinic after a normal birth and delivery following the sudden
onset of a rash involving his body. On physical exam there are numerous patches of erythema with pustules
occupying the periphery of the erythematous lesions. He appears asymptomatic at the time of visit, but the
parents report several episodes of high-grade fever that resolved without any additional sequelae. Which of the
following best explains this patient’s condition?
A. Increased localized toxins produced by Staphylococcus aureus
B. Increased circulating systemic toxins produced by Staphylococcus aureus
C. Deficiency of the Interleukin thirty-six receptor antagonist
D. None of the above is responsible for this patient's condition

Correct answer: (C) Deficiency of the Interleukin thirty-six receptor antagonist.


Explanation: The vignette details the typical onset and presentation of a child with a deficiency in the interleukin
thirty-six receptor antagonist or DITRA. This disorder can present in infants as early as one week of age and appear
similar to the von Zumbusch variant of pustular psoriasis seen in children and adults.
(A) This is the underlying pathogenesis behind Staph impetigo, bullous or non-bullous.
(B) This is the underlying pathogenesis of Staphylococcus scalded skin syndrome.

96. What is the most common cancer in POEMS syndrome?


A. Osteosclerotic myeloma
B. Castleman’s disease
C. Polycythemia
D. Plasmacytoma
E. Waldenstrom macroglobulinemia

Correct answer: (A) Osteosclerotic multiple myeloma.


Explanation: POEMS is a rare paraneoplastic syndrome characterized by polyneuropathy, organomegaly,
endocrinopathy/edema, monoclonal plasma cell proliferative disorder and skin abnormalities (including
hyperpigmentation and hypertrichosis). Although various monoclonal plasma cell proliferative disorders have been
associated with POEMS, the most common are osteosclerotic myeloma or monoclonal gammopathy of unknown
significance.The type of light chain seen in POEMS syndrome is almost always lambda. The associated heavy chain
is most commonly IgA or IgG. .Castleman’s disease, plasmacytoma and Waldenstrom macroglobulinemia occur in
only a minority of cases

97. Death of Dystrophic Epidermolysis Bullosa patients in the United States is usually due to which of the following?
A. Sepsis
B. Squamous Cell Carcinoma (SCC)
C. Hyponatremia
D. Melanoma
E. Nutritional Deficiency

45
Correct answer: (B) Squamous Cell Carcinoma (SCC).
Explanation: The most severe complication in dystrophic epidermolysis bullosa is SCC, which tends to arise in
chronically eroded skin lesions. In recessive dystrophic epidermolysis bullosa, although patients are at increased
risk of infection, the majority of patients die from metastatic SCC with SCCs often first appearing during or after
adolescence.

98. The mechanism of action of Anakinra is best described as which of the following?
A. TNF-alpha inhibitor
B. Recombinant IL-1 receptor antagonist
C. Neutrophil respiratory burst inhibitor
D. Cyclophillin-binding inhibitor of IL-2
E. Inhibitor of dihydrofolate reductase halting DNA/RNA production

Correct answer: (B) Recombinant IL-1 receptor antagonist.


Explanation: Anakinra is a synthetic recombinant IL-1 receptor antagonist utilized in the treatment of several auto-
inflammatory diseases, none the more prominent then DIRA or the Deficiency of IL-1 Receptor Antagonist. In this
disorder, there is persistent activation of the IL-1 receptor in the absence of the IL-1 receptor antagonist, leading to
a chronic pro-inflammatory state.
(A) TNF-alpha inhibitors are often used in chronic inflammatory diseases such as psoriasis, and there are several
currently available for the treatment of autoimmune and auto-inflammatory diseases. Adalimumab, a human
derived antibody to TNF-alpha, as well as etanercept, a soluble TNF-alpha receptor that blocks TNF-alpha from
binding to its receptors, are two of the common medications used in dermatologic and rheumatologic disease.
(C) Dapsone exerts its anti-inflammatory effect by inhibiting the neutrophil respiratory burst and may also prevent
chemotaxis of neutrophils.
(D) Cyclosporine exerts its anti-inflammatory effect by binding to cyclophillin leading to the dephosphorylation of
NFAT, a nuclear transcription factor, decreasing the production of IL-2 induced CD4 and CD8 T-cell stimulation.
(E) Methotrexate binds to and inhibits dihydrofolate reductase, decreasing the synthesis of purines halting DNA
and RNA production during the S-phase of the cell cycle.

99. The condition seen in the following picture is also characterized by increase in which of the following malignancy?

A. Acute Leukemia
B. Pancreatic Cancer
C. Neuroblastoma
D. Renal Cell Carcinoma
E. Testicular Cancer

Correct answer: (A) Acute Leukemia.


Explanation: Bloom Syndrome is characterized by autosomal recessive inheritance, RecQL3 helicase gene
mutations, photodistributed erythema in a butterfly distribution on nose/cheek, cheilitis, café au lait macules,

46
triangular face, prominent nose, malar hypoplasia, short stature, small mandible, and hypogonadism. Associated
malignancies include acute leukemia, lymphoma, and gastrointestinal carcinoma.

100. All of the following are caused by genetic mutations in connexin 26 EXCEPT?
A. KID Syndrome
B. Vohwinkel Syndrome
C. Bart-Pumphrey Syndrome
D. Hearing impairment
E. All of the above are associated with mutations in connexin 26

Correct answer: (E) All of the above are associated with mutations in connexin 26.
Explanation: Choices (A) (B) (C) (D) are all associated with mutations in gene responsible for the protein connexin
26. The gene is known as GJB2.
(A) KID Syndrome is characterized by keratitis, ichthyosis and deafness.
(B) Vohwinkel syndrome is a Palmoplantar keratoderma that often appears honeycombed and has features, which
include pseudoainhum of the digits (which can lead to autoamputation), star-shaped hyperkeratotic plaques on
the extensor extremities and hypkeratotic plaques located else where on the body. Like KID syndrome, the
Vohlwinkel variant associated with the GJB2 mutation, also has deafness as one of its features. It is important to
remember, however, that a Vohlwinkel syndrome variant comprised of a mutation in loricrin does not have
deafness as a feature.
(C) Bart-Pumphrey syndrome is characterized by profound hearing impairment beginning at birth, a diffuse PPK
that often appears honeycombed, and hyperkeratosis over the extensor aspects of the digits referred to as knuckle
pads. Leukonychia, which varies from each patient diagnosed with this syndrome, may improve with age.
(D) Finally, while deafness and hearing impairment are a hallmark to the three syndromes described as being
caused by mutations in the GBJ2 gene, it is important to note that hearing impairment without associated
features, or non-syndromic hearing impairment, may also be caused by mutations in connexin 26.

101. All of the following syndromes result from activating mutations in the CIAS1 gene EXCEPT?
A. Familial Cold Urticaria
B. Muckle-Wells Syndrome
C. Neonatal-onset multisystem inflammatory disease
D. Familial Mediterranean Fever
E. All of the above are due to mutations in CIAS1

Correct answer: (D) Familial Mediterranean Fever.


Explanation: Familial Mediterranean Fever (FMF) is an auto-inflammatory disease due to mutations in the MEFV
gene which encodes pyrin, and is thusly the incorrect answer. It is characterized by recurrent and painful but self-
limiting episodes of synovitis, peritonitis, and pleuritis, and occasionally involves the skin with an erysipelas-like
erythema that develops over the lower extremities. FMF is not part of the Cryopyrin Associated Periodic
Syndromes (CAPS) unlike choices (A), (B) and (C). (A) (B) (C) These are all associated with mutations in the CIAS1
gene and are categorized under the umbrella term of Cryopyrin Associated Periodic Syndromes (CAPS).
(A) Familial Cold Urticaria is an autosomal dominant disorder that is defined by painful and burning, rather than
pruritic, wheals. Its extracutaneous features include episodic pain and swelling of the joints, with accompanying
stiffness, chills and often fever after exposure to cold. The latter appears to be exacerbated with damp and windy
weather. The skin lesions are initially confined to exposed areas but later become generalized.
(B) Muckle-Wells Syndrome (MWS) shares similar features with Familial Cold Urticaria, with the added features of
secondary systemic amyloidosis and deafness. Both syndromes share the same exact genetic mutation, and may

47
be difficult to distinguish. The literature suggests that the two may be on chronological ends of a disease spectrum
but this has yet to be validated.
(C) Neonatal-onset multisystem inflammatory disease or NOMID, previously known as Chronic Infantile Neurologic
Cutaneous and Articular Syndrome or CINCA syndrome is also due to a mutation in the gene encoding cryopyrin. In
addition to its associated arthropathy and CNS manifestations, the cutaneous findings, which are often the
presenting signs are typified by a generalized but evanescent urticarial consisting of macules and papules that can
migrate during a single day and may become increasingly more severe.

102. All of the following are Major Features of Tuberous Sclerosis EXCEPT?
A. Gingival Fibromas
B. Periungual Fibromas
C. Hypomelanotic Macules
D. Facial Angiofibromas
E. Lymphangioleiomyomatosis

Correct answer: (A) Gingival Fibromas.


Explanation: Gingival fibromas are a minor feature in the diagnosis of Tuberous Sclerosis (TS). (B) (C) (D) (E) These
are all Major Diagnostic Criteria according to the Tuberous Sclerosis Complex consensus conference. Major
cutaneous features of TS include facial angiofibromas, ungual or periungual fibromas, Hypomelanotic macules and
the connective tissue nevus. Non-cutaneous major features of TS include multiple retinal nodular hamartomas,
cortical tubers, subependymal nodules, subependymal giant cell astrocytoma, renal angiomyolipoma, cardiac
rhabdomyoma, and lymphangioleiomyomatosis. Findings such as gingival fibromas and dental/enamel pits, along
with renal or bone cysts, and “confetti” hypopigmented skin lesions are all minor criteria. According to the
Consensus Conference, diagnosis of TS is either definite (with 2 Major or 1 Major plus 2 minor features), Probable
(with 1 Major and 1 minor feature), or Possible (with 1 Major or 2+ minor features).

103. Which of the following combinations of nail findings is highly suggestive of Darier Disease?
A. Longitudinal Ridging and Distal V-Nicking
B. Longitudinal Erythronychia and Subungual Hyperkeratosis
C. Distal V-Nicking and Brittle Nails
D. Longitudinal Erythronychia and Distal V-Nicking
E. Brittle Nails and Subungual Hyperkeratosis

Correct answer: (D) Longitudinal Erythronychia and Distal V-Nicking.


Explanation: The combination of longitudinal erythronychia (which presents as longitudinal red and white bands)
and distal V-nicking (secondary to brittle nails) is highly suggestive of Darier disease.
(A) (C) (E) Longitudinal ridging and fissuring can occur in Darier Disease but the combination of erythronychia and
distal V-nicking is highly suggestive for Darier Disease.
(B) Subungual hyperkeratosis is not a specific nail finding in Darier disease.

104. A 30 year old female patient presents to clinic, she has a long history of frequent epistaxis and suffers from
iron deficiency anemia. Exam is shown below.The patient should be screened for which of the following
abnormalities?
A. Pulmonary artery stenosis
B. Pulmonary capillaritis
C. Pulmonary arterovenus malformations
D. Pulmonary lymphangioleiomyomatosis
E. Pulmonary hypertension

48
Correct answer: (C) Pulmonary arterovenus malformations.
Explanation: The clinical photograph displays a patient with mucocutaneous telangiectasias, the clinical history of
epistaxis and iron deficiency anemia (likely secondary to GI bleeding). These constellation of findings is consistent
with hereditary hemorrhagic telangiectasia (HHT or Osler-Weber-Rendu syndrome). Adult patients with HHT
should be screened for pulmonary arteriovenous malformations (Choice C). Pulmonary AVMs can result in hypoxia
as arterial blood passing through these shunts cannot be oxygenated. Patients are at additional risk including
paradoxical embolic stroke and hemorrhage from rupture.
(A) Pulmonary artery stenosis is maybe a sequelae of LEOPARD syndrome (Lentigines, Ecg abnormalities, Ocular
hypertelorism, Pulmonary stenosis, Abnormalities of the genitalia, Retardation of growth, Deafness). Patients
have labial lentigines, not telegiectasias.
(B) Pulmonary capillaritis reslts from neutrophilic infiltration of the alveolar septa. A patient with recurrent
epistaxis and pulmonary capillaritis would be concerning for granulomatosis with polyangiitis (Wegener's). Clinical
findings include friable gingiva and oral ulcers plus other necrotic lesions secondary to a systemic vasculitis.
(D) Pulmonary lymphangioleiomyomatosis is the pulmonary manifestation of tuberous sclerosis. It results in cystic
lung destruction in adult females with tuberous sclerosis.
(E) Pulmonary hypertension can result from systemic sclerosis. Telangiectasias are common in systemic sclerosis
(matted/squared off telangiectasias), in this patient the additional history of anemia and epistaxis is more
consistent with HHT.

105. The following patient has a gene mutation in which of the following?

A. Steroid sulfatase
B. Transglutaminase
C. Keratin 1 and Keratin 10
D. ATP2A2
E. Emopamil-binding protein (EBP)

Correct answer: (B) Transglutaminase.


Explanation: Transglutaminase is altered in patients with Lamellar Ichthyosis and is involved in normal cross-
linking of protein and lipid envelope proteins. Key findings in newborns include a collodian membrane with
ectropion and in children/adults there is large, generalized, dark, platelike scale worse in flexures, palmoplanter
keratoderma, ectropion, scarring alopecia and nail dystrophy.
(A) Steroid sulfatase is mutated in X-linked ichthyosis.
(C) Keratin 1 and Keratin 10 are mutated in Epidermolytic Hyperkeratosis.
(D) ATP2A2 mutations are associated with Darier's disease (also called Keratosis follicularis), which is an
autosomal dominant disorder that causes warty-appearing lesions that are usually yellowish-brown in color.
(E) EBP gene mutations are found in Conradi-Hunermann Syndrome.

49
106. In PHACES syndrome, what cardiac malformation is an infant most likely to have?
A. Coarctation of aorta
B. Patent Ductus Arteriosus
C. Ventral Septal Defects
D. Atrial Septal Defects
E. Tricuspid Atresia

Correct answer: (A) Coarctation of aorta.


Explanation: Coarctation of aorta is the most common cardiac abnormality in PHACE syndrome. PHACES stands for
Posterior fossa brain malformations, large facial Hemangioma, Arterial anomalies, Cardiac anomalies and aortic
Coarctation, Eye abnormalities, and Sternal clefting and/or Supraumbilical raphe. The key cutaneous finding is a
large facial hemangioma in a V1 or V1/V2/V3 distribution. Unilateral is the more common presentation and
ulceration is frequent. The most common posterior fossa malformation is a Dandy-Walker malformation.

107. Which of the following complications has an increased risk of developing in patients with lymphatic
malformations?
A. Consumptive Coagulopathy
B. Localized Intravascular Coagulation
C. Nodularity
D. Cellulitis
E. High-Output Cardiac Failure

Correct answer: (D) Cellulitis.


Explanation: Lymphatic malformations, unlike capillary, venous, and arteriovenous malformations are more
susceptible to bacterial infections. Cellulitis is a common complication and can frequently reoccur.
(A) Consumptive coagulopathy, is a feature of disseminated intravascular coagulation, which is thought to be a
part of the Kasbach-Meritt Syndrome seen in Kaposiform Hemangioendotheliomas and Tufted Angiomas.
(B) Localized intravascular coagulation, and the formation of phleboliths, is a common complication in venous
malformations and is not seen in lymphatic malformations.
(C) Nodularity, can be a complication of long standing capillary malformations and arteriovenous malformations.
(E) High-output cardiac failure, is a feature of Stage IV arteriovenous malformations requiring intervention.

108. Nail changes in Pachyonychia Congenita are most consistent with which of the following descriptions?
A. Congenital Malalignment
B. Leukonychia
C. Koilonychia
D. Subungual Hyperkeratosis
E. Onychomadesis

Correct answer: (D) Subungual Hyperkeratosis.


Explanation: The wedge-shaped nail dystrophy in Pachyonychia Congenita (PC) is due to a thickening of the nail
bed following subungual hyperkeratosis. Upon elevation of the nail plate, it becomes thickened, brittle and may
assume extensive curvature. This can effect all 20 nails or only a few of them.
(A) Congenital malalignment of the great toes is a benign occurrence seen at birth or in the first few months of life
and presents with mild concavity or lateral displacement of the toenails of the great toe.
(B) (C) Leukonychia and koilonychia are not observed in PC.
(E) Onychomadesis, or proximal splitting of the nails, usually follows significant illness and is most commonly
associated with enteroviral infections that lead to hand foot and mouth disease.

50
109. Which of the following pairs of Keratins is found in the suprabasal layers of palmar and plantar skin?
A. Keratin 3 and 12
B. Keratin 5 and 14
C. Keratin 1 and 9
D. Keratin 1 and 10
E. Keratin 4 and 13

Correct answer: (C) Keratin 1 and 9.


Explanation: Keratins are obligate heterodimers composed of acidic and basic alpha helices that are combined to
form parallel dimers. Acidic Keratins, or Type 1 Keratins, encompass Keratins 9-28 and 31-40, while Type 2, or Basic
Keratins, is made up by Keratins 1-8 and 71-86. The final protein products of keratin heterodimers make up the
essential building blocks of the intermediate filaments found in all epithelia. Paired keratins have specificity to
different epithelia located throughout the body. Keratins 1 and 9 are located in the suprabasal layers of the palms
and soles, and mutations in these keratins can lead to diseases such as Palmoplantar Keratoderma.
(A) (B) Keratin 3 and 12 are found in the cornea, while Keratin 4 and 13 are found in non-cornified mucosa such as
oral mucous membranes.
(D) Keratin 1 and 10 is located in the suprabasal layers of epidermis not involving the palms and soles and
mutations in these keratins can cause congenital ichthyosis such as Epidermolytic Ichthyosis (also known as
Epidermolyic Hyperkeratosis or Bullous Congenital Ichthyosiform Erythroderma).
(E) Finally, Keratin 5 and 14 are found in the basal layer of all epidermal surfaces and mutations in these keratins
can lead to genetic disorders such as Epidermolysis Bullosa Simplex.

110. What is the initial treatment of porphyria cutanea tarda (PCT)?


A. Antimalarials
B. Pain control
C. Hematin and glucose loading
D. Phlebotomy
E. Beta carotene

Correct answer: (D) Phlebotomy.


Explanation: Porphyria Cutanea Tarda (PCT) has mostly autosomal dominant inheritance (UROGEN decarboxylase
gene mutation) with some sporadic/acquired cases. Key features include increased uroporphyrin in skin causing
photosensitization/erosion/skin fragility/bullae/erosions, exacerbation with alcohol ingestion, estrogen, iron
exposure, hepatic tumors, facial hypertrichosis, scarring, milia, alopecia, increased hepatocellular carcinoma, and
diabetes mellitus. Initial treatment is phlebotomy.
(A) Antimalarials are an adjunct or alternative treatment for PCT.
(B) Pain Control is usually needed in patients with acute intermittent porphyria (AIP) for peripheral neuropathy
and/or abdominal pain.
(C) Hematin and glucose loading is used to treat patients with hereditary coproporphyria (HCP).
(E) Beta carotene may be used to treat erythropoietic protoporphyria (EPP).

111. All of the following are features of mutations in keratins 6a and 16 EXCEPT?
A. Painful focal Palmoplantar Keratoderma
B. Natal Teeth
C. Hypertrophic nail dystrophy
D. Oral leukokeratosis
E. Epidermoid Cysts

51
Correct answer: (B) Natal Teeth.
Explanation: Mutations in keratin 6a and 16 are responsible for Pachyonychia Congenita Type 1 (PC-1), while
mutations in keratin 6b and 17 are responsible for Pachyonychia Congenita Type 2 (PC-2). PC-1 is also referred to
as Jadassohn-Lewadonsky type while PC-2 is called Jackson-Lawler type. Each of these types of Pachyonychia
congenital share multiple features, including those listed in choices (A), (C), (D), and (E). However, natal teeth, or
teeth present at birth, are only a feature of PC-2. Steatocystoma multiplex is more commonly seen in PC-2 and is
sometimes the only feature differentiating between the two forms of PC, however, reports of steatocystomas in
PC-1 have made natal teeth the only salient feature allowing PC-1 and PC-2 to be distinguished from one another.

112. A 5 year old child with two yellowish nodules and a history of Neurofibromatosis I is at greatest risk for which
associated disorder?
A. Gastrointestinal carcinoma
B. Vitiligo
C. Infection by encapsulated organisms
D. Juvenile myelomonocytic leukemia
E. Pyogenic granulomas

Correct answer: (D) Juvenile myelomonocytic leukemia.


Explanation: Neurofibromatosis is characterized by autosomal dominant inheritance of a gene mutation in NF-1,
involved in tumor suppression. Key features include café au lait macules, neurofibromas, axillary/inguinal freckling
(Crowe’s sign), optic glioma, Lisch nodule (iris hamartoma), sphenoid dysplasia, learning disabilities, and increased
risk nonlymphocytic childhood leukemia and seizures. Patients should have blood pressure checked to evaluate for
hypertension. In children, renal artery stenosis may be the cause. In adults, pheochromocytoma may be present.
The NIH consensus criteria for diagnosis require two or more features of the following: 6+ café au lait macules
>5mm in greatest diameter in preppubertal individuals and over 15mm in postpubertal individuals, 2+
neurofibromas or one plexiform neurofibroma, axillary or inguinal freckling, optic glioma, 2+ Lisch nodules,
osseous lesion such as sphenoid dysplasia or thinning of long bone cortex with or without pseudoarthrosis, a first-
degree relative with NF-1.
Children with JMML usually have mutations of genes involved in the RAS/MAPK signaling pathway, with
mutations in NF1, PTPN11, KRAS, NRAS, or CBL in more than 90 percent of patients with JMML. Ras signaling
proteins found in small guanosine triphosphatase (GTPase) molecules are important in cell cycle control,
proliferation, survival, and differentiation. Activation of Ras stimulates the mitogen-activated protein kinase
cascade. Neurofibromin is a GTPase activating protein that accelerates GTP hydrolysis and turns off Ras. A
mutation of NF1 interferes with deactivation of Ras and results in increased Ras activity.

113. Which of the following is a radiologic finding found in Sturge-Weber syndrome?


A. Broad thumbs
B. Calcification of ligaments
C. Enchondromas
D. Railroad tram track calcification (meningeal angiomas)
E. Polyostotic fibrous dysplasia

Correct answer: (D) Railroad tram track calcification (meningeal angiomas).


Explanation: Railroad tram track calcification (meningeal angiomas) are found in Sturge-Weber.
(A) Broad thumbs are found in Rubinstein-Taybi syndrome.
(B) Calcification of ligaments is found in pseudoxanthoma elasticum.
(C) Enchondromas are found in Maffucci syndrome.
(E) Polyostotic fibrous dysplasia is seen in McCune Albright syndrome.

52
114. This child presents with the skin lesions below, which were first noted at several weeks of age. He appears
short for his age, and has a history of recurrent respiratory infections. He is at increased risk for all of the
following, EXCEPT?

A. Leukemia
B. Squamous cell carcinomas
C. Gastrointestinal adenocarcinomas
D. Salt and pepper retinal pigmentation
E. Hypogonadism

Correct answer: (D) Salt and pepper retinal pigmentation.


Explanation: Salt and pepper retinal pigmentation is seen in Cockayne syndrome, not Bloom.
(A) Bloom Syndrome is an autosomal recessive disorder due to BLM gene mutations. Patients present with
poikiloderma on the face and extremities and recurrent infections (respiratory and GI).
(B) Patients are at increased risk of SCC formation due to decreased genomic stability.
(C) Gastric adenocarcinomas occur with higher frequency in patients with Bloom.
(E) Hypogonadism is associated.

115. A 16-year-old female patient presents to clinic with a history of acral blisters in early childhood as well as
generalized photosensitivity. The occurrence of blisters have diminished with time.On exam there is
significant atrophy of the skin overlying the dorsal hands and generalized poikiloderma on sun exposed skin.
Examination of the mucosal reveals periodontitis with poor dentition. The remainder of the examination,
including height, weight, and an ocular exam are normal. Which of the following is the most likely diagnosis?
A. Xeroderma pigmentosa
B. Bloom Syndrome
C. Kindler syndrome
D. Rothmund-Thompson syndrome
E. Cockayne syndrome

Correct answer: (C) Kindler syndrome.


Explanation: The question stem describes a patient with Kindler syndrome, an autosomal recessive disease
resulting from a loss of function mutation in kindlin-1 which functions with integrin within basal keratinocytes.
Characteristic features include acral blistering in infancy and childhood, progressive poikiloderma, skin atrophy,
photosensitivity, and gingival swelling/dental abornmalities. It is considered to be a subtype of epidermolysis
bullosa.
(A) Xeroderma pigmentosum is a genodermatosis resulting from defects in DNA repair. Like Kindler syndrome,
patients have photosensitivity and may show pokliodermatous change in sun exposed areas. Acral bullae are not
characteristic of this disease, instead patients have numerous skin cancers and may have neurological deficiencies.
(B) Bloom syndrome (congenital telangiectatic erythema), a genodermatosis resulting from a mutation in the
RECQ DNA helicase which functions in DNA repair, is characterized by erythema and telangiectasias of the malar
areas developing within the first weeks of life. Patients do not develop bullae, and short stature is a cardinal
feature of this disease.

53
(D) Rothmund-Thompson syndrome, similar to Bloom syndrome is results from mutations in RECQ DNA helicase.
Patients develop photodistributed erythema which evolves into pokilioderma and also have cataracts, short
stature and are at risk for osteosarcoma.
(E) Cockayne syndrome is another genodermatosis resulting from mutations in DNA repair enzymes. Exam findings
include a photodistributed erythema, characteristic “bird like” facies with microcephaly, large ears and a thin,
elongated nose. Other features include progressive degeneration of the nervous system, dwarfism, retinopathy
and deafness.

116. .Which of the following regarding this condition is true?

A. may occur in association with trichodiscomas


B. may occur in association with spiradenomas
C. thought to be sebaceous in origin
D. are characteristically painful
E. none of the above are associated with the
condition depicted in this case

Correct answer: (B) May occur in association with spiradenomas.


Explanation: Cylindromas can occur in association with spirademonas in Brooke-Spiegler syndrome.
(A) The condition shown above is a cylindroma, which is an adnexal tumor thought to be other either apocrine or
eccrine origin. Multiple cylindromas can coalesce to form ‘turban’ tumors. Trichodiscomas are a type of follicular
hamartoma that are associated with Birt-Hogg-Dube.
(C) Cylindromas are mostly thought to be apocrine in origin, but are sometimes interpreted as eccrine-derived.
(D) Eccrine spiradenomas are characteristically tender, not cylindromas.

117. Which of the following syndromes is not a consideration for a patient presenting with the findings
illustrated below?

A. Peutz-Jeghers
B. Laugier-Hunziker
C. Cronkhite-Canada
D. Carney Complex
E. LEOPARD Syndrome

Correct answer: (E) LEOPARD Syndrome.


Explanation: LEOPARD syndrome (lentigines, EKG abnormalities, ocular hypertelorism, pulmonic stenosis,
abnormal genitalia, retardation of growth, and sensorineural deafness), presents with dark 1-5 mm lentigines
appear anywhere on the skin, including the palms and sole, with a greater predilection for the neck and upper
trunk. These lentigines spare the oral mucosa, thus eliminating LEOPARD syndrome from the differential diagnosis
of a patient presenting with oral melanotic macules.

54
(A) (B) Laugier-Hunziker is a syndrome that demonstrates nail and oral melanotic macules and is often in the
differential for Peutz-Jeghers, the latter being characterized by mucosal melanotic macules, hamartomatous GI
polyposis, and various adenocarcinomas of the breast, GI tract, and gonads. Laugier-Hunziker has no additional
findings and is sporadic which is in contrast to the autosomal dominant inheritance seen n Peutz-Jeghers.
(C) Cronkhite-Canada syndrome presents with lentigines on the hands and feet, as well as melanotic macules on
the buccal mucosa that can occasionally be found on the vermillion border of the lips, but less commonly than in
Peutz-Jeghers or Laugier Hunziker.
(D) The Carney complex, which consists of nevi, atrial myxoma, myxoid neurofibroma, ephelides or lentigines,
atrial myxoma, mucocutaneous myxoma and blue nevi, may present with oral melanotic macules on vermillion
border of the lips, the conjunctivae, and less commonly the intraoral area.

118. Elastosis Perforans Serpiginosa is not associated with which of the following?
A. Acrogeria
B. Down Syndrome
C. Rothmund-Thomson Syndrome
D. Osteogenesis Imperfecta
E. Russell-Silver Syndrome

Correct answer: (E) Russell-Silver Syndrome.


Explanation: All of the above are associated with Elastosis Perforans Serpiginosa (EPS) except Russell-Silver
Syndrome. A mnemonic to remember EPS associations is MADPORES: Marfan’s Syndrome, Acrogeria, Down
Syndrome, Penicillamine, Osteogenesis Imperfecta, Rothmund-Thomson Syndrome, Ehlers-Danlos Syndrome and
Scleroderma.

119. Which of the following genetic mutations is INCORRECTLY paired with the disease it causes?
A. PTEN and Bannayan-Riley-Ruvalcaba Syndrome
B. ALK1 and Beckwith-Wiedemann Syndrome
C. RASA1 and Capillary Malformation- Arteriovenous Malformation Syndrome (CM-AVM)
D. PIK3CA and CLOVES syndrome
E. Tie2 and familial Venous malformation- cutaneous and mucosal Syndrome (VMCM)

Correct answer: (B) ALK1 and Beckwith-Wiedemann Syndrome.


Explanation: ALK1 is not the genetic mutation responsible for the overgrowth syndrome Beckwith-Wiedemann,
instead this mutation is responsible for Hereditary Hemorrhagic Telangiectasia or Osler-Weber-Rendu syndrome.
Biallelic overexpression of the growth factor IGF2 (Insulin Dependent Growth Factor 2) resulting from imprinting
errors, are responsible for the clinical stigmata of Beckwith-Wiedemann syndrome.
(A) (C) (D) (E) The remaining mutations listed in these choices are correctly paired to the syndromes they cause.
BRRS is associated with macrocephaly, developmental delay and penile lentigines and is due to a mutation in
PTEN. The RASA1 mutation is found in both Parkes-Weber Syndrome and the Capillary malformation –
Arteriovenous malformation syndrome (CM-AVM) and both are considered in the same spectrum of a singular
entity involving capillary malformations and fast-flow vascular malformations. PIK3CA is responsible for CLOVES
syndrome, (congenital lipomatous overgrowth, vascular malformations, epidermal nevi, and skeletal anomalies
including scoliosis), and the Macrocephaly- Capillary Malformation Syndrome (M-CM). Tie2, a tyrosine kinase
receptor involved in angiogenesis, is the mutation responsible for the familial Venous Malformation cutaneous and
mucosal syndrome (familial VMCM). This syndrome consists of small and superficial venous malformations of
predominantly the skin and mucosa with rare involvement of the GI tract, brain, and skeletal muscle. These
malformations maybe so subtle, that families are often unaware of the syndrome. Be wary of the acronym VMCM

55
in familial VMCM, do not confuse cutaneous and mucosal with capillary malformation as this syndrome does not
demonstrate the growth of capillary malformations.

120. A 2 year-old small for his age child is brought to your clinic because his parents are concerned about a skin
change when the child’s hands are submerged in water. A few minutes after performing this in clinic, you
notice pebbly changes in the child’s hands that are accompanied by edema and moderate discomfort. Which of
the following tests would be the most appropriate to perform?
A. Sweat-Chloride Test
B. Punch Biopsy
C. Iodine-Starch Test
D. Nerve Conduction Studies
E. MRI of the Spine

Correct answer: (A) Sweat-Chloride Test.


Explanation: Aquagenic Palmoplantar Keratoderma (PPK) or aquagenic wrinkling of the palms (AWP) has a known
association with Cystic Fibrosis, and the next best step to confirm this, especially in a child that had not met
growth milestones, is to perform a sweat-chloride test. The findings described in the vignette were found to be
present in up to 40-80% of Cystic Fibrosis patients. Interestingly, 25% of carriers of the ∆F508 mutation that causes
cystic fibrosis presented with AWP, while in similar studies no controls presented with this condition. Aquagenic
PPK can occur alone without cystic fibrosis and this non-disease related variant usually presents in the second
decade of life. (B) (C) (D) (E) These would not be beneficial in the work-up of this patient’s presentation and could
lead to overlooking the underlying associated disease.

121. Which of the following is not a major feature of Tuberous Sclerosis?


A. Subependymal giant cell astrocytoma
B. Dental pits
C. Cardiac rhabdomyoma
D. Renal angiomyolipoma
E. Periungual fibromas

Correct answer: (B) Dental pits.


Explanation: The following are the major and minor features of tuberous sclerosis:
Major Features:
 Hypomelanotic macules (≥3, at least 5-mm diameter)
 Angiofibromas (≥3) or fibrous cephalic plaque
 Ungual fibromas (≥2)
 Shagreen patch
 Multiple retinal hamartomas
 Cortical dysplasias*
 Subependymal nodules
 Subependymal giant cell astrocytoma
 Cardiac rhabdomyoma
 Lymphangioleiomyomatosis (LAM)†
 Angiomyolipomas (≥2)†
Minor features
 “Confetti” skin lesions
 Dental enamel pits (>3)

56
 Intraoral fibromas (≥2)
 Retinal achromic patch
 Multiple renal cysts
 Nonrenal hamartoma

122. A patient presents to your clinic requesting removal of numerous pink telangiectatic papules in the midline
portion of her face. On clinical exam you notice hypopigmented macules on her trunk, which she has had
since birth, as well as two dense, periungual skin colored papules on her R and L great toes? What percentage
of this patient’s offspring would present with her disease?
A. 25% of male offspring
B. 25% of female offspring
C. 100% of female offspring
D. 100% of male offspring
E. 50% of all offspring

Correct answer: (E) 50% of all offspring.


Explanation: The inheritance of Tuberous Sclerosis (TS) was initially described as an autosomal dominant disease.
In offspring of parents with an autosomal dominant disease there is an inheritance pattern of 50% since only one
mutation from either the maternal or paternal donor is required for the manifestation of a disease. All offspring
who carry the mutated gene will present with manifestations of the disease, however, like most genetic diseases, a
significant component of any given patient presentation is also dependent on the penetrance and expressivity of
that particular disease.
Despite TS being autosomal dominant in its inheritance pattern, a majority of cases presenting today have been
reported in individuals with sporadic mutations. While a new mutation can present in a patient with no known
family history of TS, its inheritance to that patients children would follow an autosomal dominant pattern. It is
important not to confuse sporadic mutations with a pattern of inheritance. Sporadic mutations can occur in
anyone, however, if the inheritance pattern of that disease is known, then offspring of patients who have acquired
the mutation will be affected accordingly.
In autosomal recessive diseases, both male and females are affected equally, and both can equally transmit to
their offspring. All children of two affected individuals are affected. If parents are heterozygous, the risk of disease
is 25% and 2/3 of the healthy siblings will be heterozygous carriers of the gene mutation but will not express the
phenotypical characteristics of the disease. (Choice A and B) In X-linked Dominant Diseases, 100% of female
offspring will have the disease (Choice C), while male offspring typically die in utero as these lethal mutations tend
to be deleterious during embryogenesis. Finally, in X-linked Recessive Diseases, all male offspring will have the
disease as they have only one X-chromosome (Choice D), while all female offspring of male carriers with an X-
linked recessive disease will carry the disease increasing the risk of passing it on to their children. Female carriers
of the disease may also have mild manifestations of the inherited disease, but not to the full effect of their
brothers. Of note, there is no male-to-male transmission in XLR diseases.

123. Which of the following aberrant proteins is INCORRECTLY matched to its associated syndrome?
A. IL-36 Receptor Antagonist – Pustular Psoriasis
B. Cryopyrin – Muckle-Wells Syndrome
C. PSTPIP1 – PAPA Syndrome
D. Pyrin – Deficiency of the IL-1 Receptor Antagonist
E. P55 TNF Receptor – TRAPS

57
Correct answer: (D) Pyrin – Deficiency of the IL-1 Receptor Antagonist.
Explanation: All of the above pairs of mutated proteins and their consequential outcomes are matched except for
choice (D). Mutations in Pyrin are responsible for Familial Mediterranean Fever (FMF) and not the deficiency in IL-1
Receptor Antagonist or DIRA. The mutated IL-1 Receptor antagonist is what gives rise to DIRA, a syndrome
characterized by fetal distress, joint swelling, osteopenia, lytic bone lesions, thrombosis, and pustulosis.
(A) The IL-36 Receptor Antagonist deficiency (DITRA) is a known cause of pustular psoriasis. In addition to
erythematous patches studded with pustules with central clearing and exfoliation, this condition is also
characterized by high fevers and malaise.
(C) PAPA syndrome stands for pyogenic (sterile, destructive) arthritis, pyoderma gangrenosum, acne and myositis
and is caused by mutations in the PSTPIP1 gene and protein.
(E) TRAPS or TNF-receptor associated periodic syndrome and is characterized by recurrent and sometimes
prolonged (2-3 weeks), serositis, rash, conjunctivitis, and arthritis, and is due to mutations in the P55 TNF receptor
protein.

124. All of the following are hereditary disorders that feature palmoplantar pits EXCEPT?
A. Basal Cell Nevus Syndrome
B. Darier Disease
C. Cowden Disease
D. Reticulate Acropigmentation of Kitamura
E. Pitted Keratolysis

Correct answer: (E) Pitted Keratolysis.


Explanation: Pitted Keratolysis presents as moist and macerated plantar pits that are predominantly located on
the pressure bearing areas of the plantar surfaces. They are often associated with malodor and are due to the
corynebacterium Micrococcus sedentaris. Factors that increase the risk of acquiring this condition include
hyperhidrosis, poor hygiene, and walking barefoot. Treatment is with topical antibiotics such as Erythromycin or
Clindamycin. All other choices (A-D) can present with palmar and plantar pits, but are genetically acquired.
(A) Basal cell nevus syndrome is an autosomal dominant mutation in the PATCHED1 gene (a tumor suppressor
gene involved in hedgehog signaling by inhibiting smoothened). Clinical features include basal cell carcinomas,
medulloblastomas, pits of the palms and soles and odontogenic cysts of the jaw.
(B) Darier disease is an autosomal dominant mutation in the endoplastic reticulum Ca2+ pump (ATP2A2). Clinical
features include yellow/brown foul smelling greasy papules in seborrheic areas, red and white longitudinal nail
bands ending in V-shaped nicks, and pits on the palms and soles.
(C) Cowden disease is an autosomal dominant mutation in PTEN. Clinical features include trichilemmomas,
mucosal papillomas, palmoplantar keratosis or pits and pigmented macules on the glans penis. Patients are at risk
for breast, thyroid malignancy and GI hamartomas.
(D) Reticulate acropigmetnation of kitamura is a rare atusomal dominant syndrome characterized by reticulate,
freckle like hyerpigmentation on the hands that then generalizes to the rest of the body. Pits may be found on the
palms and soles.

58
125. A patient known to your clinic presents with the sudden onset of pain in a blue nodule located on the right
arm. He has had this nodule for several years without any complications. It is soft, compressible, and pulseless
and prior to his presentation today was painless. Which of the following lab values is consistent with the
patient’s current findings?
A. High levels of fibrinogen
B. Thrombocytopenia
C. Elevated D-Dimers
D. Increased clotting factors
E. Elevated troponins

Correct answer: (C) Elevated D-Dimers.


Explanation: Venous malformations are at increased risk of developing localized intravascular coagulation or LIC.
Like smaller clots in superficial venous malformations that give rise to phleboliths, LIC typically presents with the
sudden onset of new pain in what are normally asymptomatic venous malformations. It is more common in larger
venous malformations, and those that are intramuscular. In addition to its thrombotic presentation following LIC,
it can also present with bleeding. These tend to persist through life. Localized intravascular coagulation is a
process consistent with consumptive coagulopathy, and is characterized by lab findings demonstrating a decrease
in clotting factors (D), low levels of fibrinogen (A), and elevated D-Dimers, making choice (C) the correct answer.
Thrombocytopenia (B) is not a feature of the localized intravenous coagulation that takes place in venous
malformations. This differentiates the consumptive coagulopathy in LIC from that which takes place in the
Kasbach-Meritt Syndrome. Troponins are not elevated in localized intravascular coagulation.

126. All of the statements regarding cutis marmorata telangiectata congenita (CMTC) are true EXCEPT?
A. Lesions of CMTC can involve the entire surface of the body.
B. Unlike other vascular malformations, CMTC is not associated with body asymmetry.
C. CMTC can present with focal areas of ulceration and/or atrophy.
D. CMTC can sometimes be associated with alopecia and syndactyly.
E. Congenital Hypothyroidism should be screened for in newborns with CMTC.

Correct answer: (B) Unlike other vascular malformations, CMTC is not associated with body asymmetry.
Explanation: One of the most common associated and frequently described anomalies with CMTC is body
asymmetry. Limb girth discrepancies compose the majority of these associations, and unlike other vascular
malformation, associated limb abnormalities in CMTC extremities are shorter and thinner from having less fat,
diminished muscles or bones. (A) (C) (D) (E) These are all true statements regarding CMTC. Lesions of CMTC can be
localized, segmental, or diffuse involving large portions of the body. Parts of the cutaneous features of CMTC,
aside from the reticulated vascular network, include atrophy and ulceration of the areas of the skin that are in
between the vascular network. This occurs in greater frequency over the limbs. In Adams-Oliver Syndrome, CMTC is
associated with syndactyly, cardiac defects, and alopecia. Along with aplasia cutis, CMTC is often an early finding
in this particular syndrome. Finally, congenital hypothyroidism, while not a salient feature of CMTC, has been
reported in increasing frequency in CMTC, and warrants laboratory evaluation.

127. Gap junctions are cell membrane proteins that serve as channels of communication between the cytoplasm of
neighboring cells. Which of the following proteins is necessary for the physiologic properties of gap junctions?
A. Cathepsin C
B. Loricrin
C. Keratin
D. Connexin
E. Desmoglein

59
Correct answer: (D) Connexin.
Explanation: Gap junctions are composed membrane proteins called connexins. Six connexins compose one helical
channel called a connexon. Connexons of one cell contact connexons from a neighboring cell allowing for the
formation of a cytoplasm-to-cytoplasm channel between adjacent cells.
(A) Cathepsin C is an agonist of serine proteases found in inflammatory cells. It is mutated in both Papillon-Lefèvre
Syndrome and Haim-Munk Syndrome.
(B) Loricrin is the major component of the cornified cell envelope in epidermal keratinocytes that compose the
granular layer initially, and upon maturation make up the stratum corneum. It is mutated in the Vohlwinkel
Syndrome variant that is characterized by ichthyosis without deafness.
(C) Keratins dimerize to form intermediate filaments and make up the cytoskeleton of keratinocytes. They
associated with a number of cytoplasmic as well as transmembrane proteins in both desmosomes and
hemidesmosomes. There are numerous types of keratin and some are specific to certain types of epithelia. Keratin
mutations are the cause of numerous genodermatoses including epidermolysis bullosa simplex, epidermolytic as
well as non-epidermolytic palmoplantar keratoderma, and epidermolytic ichthyosis (also known as epidermolytic
hyperkeratosis or bullous congenital Ichthyosiform erythroderma).
(E) Desmoglein, along with desmocollin, are cadherins that function as cell-cell adhesion mediators and form the
core components of desmosomes. Mutations in desmoglein 1 are responsible for Striate PPK.

128. A six year-old boy, who has been wheelchair-bound since the age of 2, is brought to your clinic by his mother.
The father is concerned about numerous blood vessels appearing on his sons skin. In addition to discussing the
possible diagnosis you inform the mother that she as a possible carrier is at increased risk of which malignancy?
A. Thyroid Cancer
B. Hepatic Cancer
C. Pancreatic Cancer
D. Breast Cancer
E. Renal Cancer

Correct answer: (D) Breast Cancer.


Explanation: While patients affected with Ataxia-Telangiectasia (AT) have a 70-fold higher likelihood of developing
leukemia, and a 200-fold higher risk of developing lymphoma when compared to the normal population,
heterozygous carriers of the genetic mutation responsible for AT, have a two- to threefold increase in risk of
developing breast cancer. Additionally, heterozygous carriers of the ATM mutation, have a higher risk of death
from cancer, with higher mortality in individuals under 50 years of age. This includes breast cancer and less
frequently stomach, colon, and lung cancer. It is important to note that difficulty walking due to cerebellar ataxia
has an earlier onset compared to the onset of cutaneous telangiectasias in AT, however the exception to this rule is
ocular telangiectasias which may occur as early as 1 year of age. (A) (B) (C) (E) These are incorrect as they are not
found to have an increased incidence in heterozygotic carriers of the ATM gene.

60

You might also like